Крок 2 - Медицина 2017 (буклет)

1 / 200
До ревматологічного відділення надійшла хвора 20-ти років з явищами поліартриту. На шкірі обличчя - почервоніння у вигляді ’’метелика” У сечі: білок - 4,8 г/л. Лікар припустив наявність у хворої системного червоного вовчаку. Який додатковий метод дослідження є найбільш інформативним для верифікації діагнозу? A 20-year-old patient with symptoms of polyarthritis was admitted to the rheumatology department. On the skin of the face - redness in the form of a 'butterfly' In the urine: protein - 4.8 g /l. The doctor assumed that the patient had systemic lupus erythematosus. What additional research method is the most informative for verifying the diagnosis?

Загальний аналіз крові General blood test

Аналіз крові на LE-клітини Blood analysis for LE cells

Аналіз крові на антинуклеарні антитіла Blood analysis for antinuclear antibodies

Імунологічне дослідження крові Immunological examination of blood

Аналіз крові на ревматоїдний фактор Blood analysis for rheumatoid factor

2 / 200
Після вживання яєчні, приготованої вдома з яєць качки, у хворих через 8 годин виникли наступні симптоми захворювання: температура - 39oC, головний біль, блювання, біль у животі, згодом - діарея. Випорожнення часті, зі слизом, смердючі. Тривалість захворювання - 3 доби. Яка хвороба мікробної природи має місце? After eating scrambled eggs prepared at home from duck eggs, patients developed the following symptoms after 8 hours: temperature - 39oC, headache, vomiting, abdominal pain, later - diarrhea. Excretions are frequent, with mucus, smelly. The duration of the disease is 3 days. What microbial disease is occurring?

Бруцельоз Brucellosis

Харчовий ботулізм Food botulism

Сальмонельоз Salmonellosis

Туляремія Tularemia

Сибірка Anthrax

3 / 200
У хворої на 4-у добу після оперативного втручання з приводу гнійного маститу загальний стан погіршився, зросла температура до 40oC, частота пульсу - 110/хв., частота дихання - 24/хв. Артеріальний тиск - 100/60 мм рт.ст. В аналізі крові кількість лейкоцитів - 18 • 109 /л, кількість паличкоядерних форм - 19%. Яке ускладнення можна припустити? On the 4th day after surgery for purulent mastitis, the patient's general condition worsened, the temperature rose to 40oC, the pulse rate was 110/min., the respiratory rate - 24/min. Blood pressure - 100/60 mm Hg. In the blood test, the number of leukocytes - 18 • 109 /l, the number of rod-nuclear forms - 19%. What complications can be assumed?

Гостре респіраторне захворювання Acute respiratory disease

Перитоніт Peritonitis

Нагноєння рани Wound suppuration

Сепсис Sepsis

Газова гангрена Gas gangrene

4 / 200
У хворої вранці з’явилися нудота, одноразове блювання, сухість у роті. Ввечері зазначила двоїння предметів, ’туман” перед очима, похлинання під час ковтання рідкої їжі. Об’єктивно: температура тіла - 36,4oC, птоз, мідріаз, анізокорія, відсутність блювотного і ковтального рефлексів, сухість слизових. З боку внутрішніх органів відхилень не виявлено. Яке захворювання найбільш імовірне? In the morning, the patient developed nausea, one-time vomiting, dry mouth. In the evening, she noted double vision of objects, 'fog' in front of her eyes, regurgitation when swallowing liquid food. Objectively: body temperature - 36.4oC, ptosis, mydriasis, anisocoria, absence of vomiting and swallowing reflexes, dryness of the mucous membranes. No abnormalities were found on the part of the internal organs. What disease is most likely?

Туберкульозний менінгоенцефаліт Tuberculous meningoencephalitis

Ботулізм Botulism

Гостре порушення мозкового кровообігу Acute cerebrovascular accident

Стовбуровий енцефаліт Stem encephalitis

Вірусний менінгоенцефаліт Viral meningoencephalitis

5 / 200
У дівчинки 15-ти років 2 роки тому діагностований хронічний холецистохолангіт. Дієти не дотримувалася. Стан погіршився останні 3 місяці. Відзначається підвищення температури тіла. Болі у животі нападоподібного характеру після жирної, гострої їжі. Непокоїть свербіння шкіри. Язик обкладений білим нальотом. Живіт м’який, печінка +3 см, пальпація болісна, позитивні міхурні симптоми. У крові: лейкоцити -12109/л, швидкість осідання еритроцитів -20 мм/год. Який препарат слід обов’язково включити до комплексу лікувальних заходів? A 15-year-old girl was diagnosed with chronic cholecystocholangitis 2 years ago. She did not follow a diet. The condition has worsened over the past 3 months. There is an increase in body temperature. Abdominal pains of an attack-like nature after fatty, spicy food. Itchy skin bothers. The tongue is covered with a white coating. Abdomen is soft, liver +3 cm, palpation is painful, positive bladder symptoms. In the blood: leukocytes -12109/l, erythrocyte sedimentation rate -20 mm/h. What drug must be included in the complex of treatment measures?

Ентеросорбенти Enterosorbents

Ферменти підшлункової залози Pancreatic enzymes

Прокінетики Prokinetics

Антибіотики Antibiotics

Жовчогінний Cholesterol

6 / 200
До жіночої консультації звернулася жінка 40-ка років зі скаргами на порушення менструального циклу за типом гіперпо-ліменореї протягом півроку, тягнучі болі в нижніх відділах живота, слабкість. В ході гінекологічного обстеження тіло матки збільшене до 12 тижнів вагітності, щільне, рухоме, безболісне. В крові: Hb- 90 г/л. Яку патологію можна припустити? A 40-year-old woman came to a women's consultation with complaints of menstrual cycle disturbances in the form of hyperpolymenorrhea for six months, pulling pains in the lower abdomen, weakness. In during a gynecological examination, the body of the uterus is enlarged up to 12 weeks of pregnancy, dense, mobile, painless. In the blood: Hb - 90 g/l. What pathology can be assumed?

Кістома яєчника Ovarian cystoma

Вагітність Pregnancy

Міома матки Uterine myoma

Дисфункціональна маткова кровотеча Dysfunctional uterine bleeding

Рак тіла матки Cancer of the uterine body

7 / 200
Повторнонароджуюча 26-ти років прибула до пологового відділення у зв’язку з вагітністю 40 тижнів та початком пологової діяльності. 2 години тому відійшли навколоплідні води. Положення плоду поздовжнє, головне передлежання. ОЖ- 100 см, ВДМ- 42 см. Перейми кожні 4-5 хвилин, протягом 25-ти секунд. Під час внутрішнього акушерського обстеження: шийка матки згладжена, відкриття 4 см. Плідного міхура немає. Іоловка плоду притиснута до входу в малий таз. Яке ускладнення виникло в пологах? A 26-year-old woman who gave birth again came to the maternity ward due to her 40-week pregnancy and the beginning of labor. 2 hours ago, the amniotic fluid broke. The position of the fetus is longitudinal, the main presentation. OZ- 100 cm, VDM- 42 cm. Take every 4-5 minutes, for 25 seconds. During the internal obstetric examination: the cervix is smoothed, the opening is 4 cm. There is no amniotic sac. The fetal isola is pressed against the entrance to the small pelvis. What complication occurred during childbirth?

Вторинна слабкість пологової діяльності Secondary weakness of labor activity

Дискоординована пологова діяльність Discoordinated birth activity

Первинна слабкість пологової діяльності Primary weakness of labor activity

Клінічно вузький таз Clinically narrow pelvis

Передчасне вилиття навколоплідних вод Premature discharge of amniotic fluid

8 / 200
Пацієнтка 37-ми років хворіє більше 10-ти років. Розповідає, що її мозком вже давно заволоділи 'злочинні вчені-фізики” які випробовують на ній різні типи психотропної зброї. Відчуває на собі вплив лазерних променів, постійно чує повідомлення, які передаються їй безпосередньо в мозок. Емоційно монотонна, майже цілі дні проводить у ліжку, робить якісь записи, які нікому не показує. Свідомість ясна, формальних інтелектуально-мнестичних порушень не виявлено. Який діагноз у даної хворої'? A 37-year-old patient has been sick for more than 10 years. She says that her brain has long been taken over by 'criminal physicists' who are testing various types of psychotropic drugs on her weapons. Feels the effect of laser beams on herself, constantly hears messages that are transmitted directly to her brain. Emotionally monotonous, spends almost whole days in bed, makes some notes that she does not show to anyone. Consciousness is clear, no formal intellectual-mnestic disorders were detected. What is the diagnosis of this patient?

Реактивний параноїд Reactive Paranoid

Хронічний маячний розлад Chronic delusional disorder

Шизо-афективний психоз Schizo-affective psychosis

Параноїдна шизофренія Paranoid schizophrenia

Інволюційний параноїд Involutional Paranoid

9 / 200
Дівчина 13-ти років протягом 5-ти років скаржиться на біль у правому підребер’ї, що віддає у праву лопатку, напади болю пов’язані з порушенням дієти, вони нетривалі, легко знімаються спазмолітичними засобами. Під час нападу болю пальпація живота болісна, максимально в точці проекції жовчного міхура. З найбільшою імовірністю у хворої має місце: A 13-year-old girl has been complaining of pain in the right hypochondrium for 5 years, radiating to the right shoulder blade, pain attacks are associated with a diet violation , they are short-lived, easily removed with antispasmodic agents. During a pain attack, palpation of the abdomen is painful, maximally at the point of projection of the gallbladder. The patient is most likely to have:

Дискінезія жовчовивідних шляхів Biliary tract dyskinesia

Хронічний холецистит Chronic cholecystitis

Виразкова хвороба 12-палої кишки Duodenal ulcer disease

Хронічний гастродуоденіт Chronic gastroduodenitis

Хронічний панкреатит Chronic pancreatitis

10 / 200
В ході медичного огляду у хворого було виявлено: затримка росту, гіпогонадизм, статевий інфантилізм, відсутність вторинних статевих ознак, збільшення печінки та селезінки. Це свідчить про недостатність у харчуванні такого елементу: During the medical examination, the patient was found to have: growth retardation, hypogonadism, sexual infantilism, absence of secondary sexual characteristics, enlargement of the liver and spleen. This indicates insufficient nutrition of such an element:

Йод Iodine

Цинк Zinc

Мідь Copper

Кальцій Calcium

Вітамін D Vitamin D

11 / 200
До дільничного терапевта звернувся хворий 37-ми років, який внаслідок загострення хронічного обструктивного бронхіту протягом року був тимчасово непрацездатним 117 днів. Визначте юридично вірну тактику лікаря в даному випадку: A 37-year-old patient who, due to an exacerbation of chronic obstructive bronchitis during the year, was temporarily unable to work for 117 days turned to the district therapist. Determine the legally correct tactics of the doctor in this case:

Направлення хворого на санаторно-курортне лікування Referral of the patient to sanatorium-resort treatment

Продовження листка непрацездатності терапевтом Continuation of sick leave by therapist

Направлення на лікарсько-консультаційну комісію для продовження листка непрацездатності Referral to the medical advisory commission for the extension of the sick leave certificate

Хворий підлягає направленню на медико-соціальну експертну комісію The patient is subject to referral to the medical and social expert commission

Видача трудового (доплатного) листка непрацездатності Issue of labor (paid) sick leave

12 / 200
Впершенароджуюча 30-ти років. Голівка у порожнині малого тазу. Серцебиття плоду почало уповільнюватися, з’явилася аритмія. Що слід застосувати? 30-year-old woman giving birth for the first time. The head is in the pelvic cavity. The fetal heartbeat began to slow down, an arrhythmia appeared. What should be applied?

Порожнинні акушерські щипці Hollow obstetric forceps

Вихідні акушерські щипці Outgoing obstetric forceps

Перінеотомію Perineotomy

Кесарів розтин Caesarean section

Шкірно-головні щипці Scalp forceps

13 / 200
Хворий 20-ти років скаржиться на періодичне запаморочення, головний біль, носові кровотечі, ниючий біль в ділянці серця, судоми м’язів ніг, похолодання ступнів. Об’єктивно: фізичний розвиток супроводжується диспропорцією м’язової системи (м’язи верхньої половини тіла гіпертрофо-вані, при відносній гіпотрофії м’язів тазу та нижніх кінцівок), ноги на дотик холодні. Пульс - 86/хв., ритмічний, артеріальний тиск: на руках - 200/100 мм рт.ст., на ногах -140/90 мм рт.ст. Яке захворювання можна припустити в першу чергу? A 20-year-old patient complains of periodic dizziness, headache, nosebleeds, aching pain in the heart area, leg muscle cramps, coldness of the feet. Ob' objectively: physical development is accompanied by a disproportion of the muscular system (the muscles of the upper half of the body are hypertrophied, with relative hypotrophy of the muscles of the pelvis and lower extremities), the legs are cold to the touch. Pulse - 86/min., rhythmic, blood pressure: on the hands - 200/100 mm Hg, on the legs - 140/90 mm Hg. What disease can be assumed first of all?

Нейроциркуляторна дистонія Neurocirculatory dystonia

Гіпертонічна хвороба Hypertensive disease

Облітеративний ендартеріїт Endarteritis obliterative

Коарктація аорти Coarctation of the aorta

Хвороба Такаясу Takayasu disease

14 / 200
Пацієнтка 46-ти років хворіє на цукровий діабет 9 років, отримує інсулін Моно-тард 26 од. вранці і 18 од. ввечері. Скарги на слабість, млявість вранці після сну, головний біль, пітливість вночі. Об’єктивно: пульс - 72/хв., артеріальний тиск - 125/70 мм рт.ст. Межі серця в нормі. Печінка +4 см. Цукор крові: 8:00 - 14 ммоль/л, 12:00 -9 ммоль/л, 17:00 - 11 ммоль/л. Цукор сечі в межах 0,5-1%. З чим найбільш імовірно пов’язані скарги, що виникають у хворої вночі? A 46-year-old patient has been suffering from diabetes for 9 years, receives Mono-tard insulin 26 units in the morning and 18 units in the evening. Complains of weakness, lethargy in the morning after sleep, headache, sweating at night. Objectively: pulse - 72/min., blood pressure - 125/70 mm Hg. Heart boundaries are normal. Liver +4 cm. Blood sugar: 8:00 - 14 mmol /l, 12:00 -9 mmol/l, 17:00 - 11 mmol/l. Urine sugar in the range of 0.5-1%. What is the most likely cause of the patient's complaints at night?

Недостатність дози інсуліну вранці Insufficient dose of insulin in the morning

Надлишок дози інсуліну ввечері Overdose of insulin in the evening

Наявність гепатозу Presence of hepatosis

Клімактеричний синдром Climacteric syndrome

Недостатня вечірня доза інсуліну Insufficient evening dose of insulin

15 / 200
Хворий 43-х років скаржиться на кашель з виділенням невеликої кількості безбарвного харкотиння, болі в правій половині грудної клітки при диханні, задишку, підвищення температури до 39oC. Захворів гостро. Приймав аспірин. Об’єктивно: на губах герпес. В проекції нижньої частки правої легені - притуплення перкуторного звуку, посилення голосового тремтіння, бронхіальне дихання. Рентгенологічно: гомогенне затемнення частки з чіткими контурами. Яка етіологія пневмонії найбільш імовірна? A 43-year-old patient complains of a cough with a small amount of colorless sputum, pain in the right half of the chest when breathing, shortness of breath, temperature rise to 39oC. He became acutely ill . Took aspirin. Objectively: herpes on the lips. In the projection of the lower lobe of the right lung - dulling of the percussion sound, increased vocal tremor, bronchial breathing. Radiologically: homogeneous darkening of the lobe with clear contours. What is the most likely etiology of pneumonia?

Staphylococcus pneumoniae Staphylococcus pneumoniae

Legionella pneumoniae Legionella pneumoniae

Klebsiella pneumoniae Klebsiella pneumoniae

Streptococcus pneumoniae Streptococcus pneumoniae

Mycoplasma pneumoniae Mycoplasma pneumoniae

16 / 200
Хвора 24-х років звернулася до гінеколога зі скаргами на появу виростів у ділянці статевих органів. Оглянувши хвору, лікар виявив на великих і малих соромітних губах сосочкоподібні розростання, що нагадують цвітну капусту, м’якої консистенції, неболючі, неерозовані. Хвору направлено на консультацію до дерматолога. Який діагноз найбільш імовірний? A 24-year-old patient turned to a gynecologist with complaints about the appearance of growths in the genital area. After examining the patient, the doctor found papilla-like growths on the labia majora and labia minora, which resemble cauliflower, soft consistency, painless, non-erosive. The patient was sent for a consultation with a dermatologist. What is the most likely diagnosis?

Папіломатоз Papillomatosis

Гострокінцеві кондиломи Condyloma acuminate

Вегетуюча пухирчатка Vegetable pemphigus

Широкі кондиломи Broad condylomas

Гранульоматозний кандидоз Granulomatous candidiasis

17 / 200
Хлопчик 3-х місяців госпiталiзований у зв’язку з затяжною жовтяницею і постійними закрепами. Хворий з дня народження. Вагітність у матері була ускладнена токсикозом. Під час огляду мало активний, обличчя набрякле, макроглосія, шкіра іктерична. Вузькі очні щілини. М’язовий тонус знижений. Брадикардія. Який найбільш імовірний діагноз? A 3-month-old boy is hospitalized due to protracted jaundice and constant constipation. He has been sick since his birth. The mother's pregnancy was complicated by toxicosis. During the examination, little active, swollen face, macroglossia, icteric skin. Narrow eye slits. Decreased muscle tone. Bradycardia. What is the most likely diagnosis?

Муковісцидоз Cystic Fibrosis

Рахіт Rickets

Хвороба Дауна Down's disease

Хвороба Гіршпрунга Hirschsprung's disease

Вроджений пілоростеноз Congenital pylorostenosis

18 / 200
Хвора 21-го року без постійного місця проживання скаржиться на послаблення випорожнень впродовж 2-х місяців, втрату ваги на 13 кг, слабкість, постійну субфебрильну температуру, рецидивуючий герпес. На губах герпетичні висипання, генералізована лімфаденопатія, збільшення печінки до 2 см. У крові: еритроцити - 4,4 • 1012/л, Hb- 115 мг/л, швидкість осідання еритроцитів -15 мм/год, лейкоцити -10,0 • 109 /л, е.- 2%, п.- 6%, с.- 61%, л.-17%, м.- 3%. Атипові мононуклеари - 6%. Який найбільш імовірний діагноз? A 21-year-old patient without permanent residence complains of loose bowel movements for 2 months, weight loss of 13 kg, weakness, constant low-grade fever, recurrent herpes Herpetic rashes on the lips, generalized lymphadenopathy, enlargement of the liver up to 2 cm. In the blood: erythrocytes - 4.4 • 1012/l, Hb - 115 mg/l, erythrocyte sedimentation rate -15 mm/h, leukocytes -10.0 • 109 /l, e.- 2%, p.- 6%, p.- 61%, l.-17%, m.- 3%. Atypical mononuclear cells - 6%. What is the most likely diagnosis?

ВІЛ-інфекція HIV infection

Кишковий амебіаз Intestinal amoebiasis

Лімфогранулематоз Lymphogranulomatosis

Інфекційний мононуклеоз Infectious mononucleosis

Сепсис Sepsis

19 / 200
Хвора 64-х років скаржиться на загальну слабкість, шум у голові, осиплість голосу. Об’єктивно: блідість з жовтяничним відтінком, язик червоний зі згладженими сосочками, асиметрія тактильної і больової чутливості. Пульс - 120/хв., артеріальний тиск - 80/50 мм рт.ст. Пальпується селезінка. У крові: НЬ- 58 г/л, еритроцити -1,24*1012/л, лейкоцити - 2,8-109 /л, тромбоцити - 140 * 109 /л, швидкість осідання еритроцитів - 17 мм/год, анізоцитоз, пойкілоцитоз - виражені (++). Яке дослідження буде вирішальним у з’ясуванні генезу анемії? A 64-year-old patient complains of general weakness, noise in the head, hoarseness of voice. Objectively: paleness with a jaundiced tint, red tongue with flattened papillae, asymmetry tactile and pain sensitivity. Pulse - 120/min., blood pressure - 80/50 mm Hg. The spleen is palpable. In the blood: Hb - 58 g/l, erythrocytes - 1.24*1012/l, leukocytes - 2, 8-109 /l, platelets - 140 * 109 /l, erythrocyte sedimentation rate - 17 mm/h, anisocytosis, poikilocytosis - pronounced (++). Which study will be decisive in clarifying the genesis of anemia?

Непряма проба Кумбса Indirect Coombs test

Стернальна пункція Sternal puncture

Люмбальна пункція Lumbar puncture

Пряма проба Кумбса Direct Coombs test

Фіброгастроскопія Fibrogastroscopy

20 / 200
Хворий 20-ти років доставлений бригадою швидкої медичної допомоги в коматозному стані. Зіниці різко звужені, ціаноз і сухість слизових оболонок, шкіра обличчя і тулуба бліда, волога і холодна. Пульс -60/хв., артеріальний тиск - 80/60 мм рт.ст., частота дихання - 8-10/хв. Який найбільш імовірний діагноз? A 20-year-old patient was delivered by an ambulance in a comatose state. The pupils are sharply narrowed, cyanosis and dryness of the mucous membranes, the skin of the face and body is pale, moist and cold Pulse - 60/min., blood pressure - 80/60 mm Hg, respiratory rate - 8-10/min. What is the most likely diagnosis?

Отруєння сурогатами алкоголю Alcohol surrogate poisoning

Отруєння бензодіазепіновими похідними Poisoning with benzodiazepine derivatives

Отруєння опіатами Opiate poisoning

Отруєння барбітуратами Barbiturate poisoning

Отруєння атропіном Atropine poisoning

21 / 200
Внаслідок багаторічного забруднення навколишнього середовища викидами металургійного заводу на прилеглій житловій території міста сформувалася біогеохімічна провінція. За якою провідною ознакою можна надати їй гігієнічну оцінку? As a result of long-term pollution of the environment by emissions from a metallurgical plant, a biogeochemical province was formed in the adjacent residential area of the city. By what leading feature can it be given a hygienic assessment?

Погіршення якості продуктів рослинного походження Deterioration in the quality of plant products

Забруднення атмосферного повітря Atmospheric air pollution

Підвищений рівень захворюваності населення Increased population morbidity

Накопичення в ґрунті специфічних хімічних речовин Soil accumulation of specific chemicals

Обмеження умов санітарно-побутового водокористування Restriction of conditions of sanitary and domestic water use

22 / 200
Жінка 35-ти років скаржиться на біль у ділянці серця (”щемить” ’’свердлить”), що виникає переважно у ранкові години в осінньо-весняний період, з іррадіацією болю в шию, спину, живіт; часте серцебиття, а також зниження загального життєвого тонусу. Виникнення цього стану не пов’язане з фізичним навантаженням. Увечері стан поліпшується. Соматичний, неврологічний статус та ЕКГ - без патології. Яка найбільш імовірна патологія зумовила таку клінічну картину? A 35-year-old woman complains of pain in the area of the heart ('pinching' 'drilling'), which occurs mainly in the morning hours in the autumn-spring period, with pain radiating to the neck, back, abdomen; frequent heartbeat, as well as a decrease in general vitality. The occurrence of this condition is not related to physical exertion. In the evening, the condition improves. Somatic, neurological status and ECG - without pathology. What is the most likely pathology that caused such a clinical picture?

Соматизована депресія Somatized depression

Стенокардія спокою Angina at rest

Нейроциркуляторна дистонія Neurocirculatory dystonia

Неврозоподібна шизофренія Neurosis-like schizophrenia

Іпохондрична депресія Hypochondriac depression

23 / 200
Хвора 18-ти років звернулася до лікаря у зв’язку з вираженою слабкістю, лихоманкою, прогресивним схудненням, запамороченням. Об’єктивно: при зрості 165 см, вага - 40 кг, шкіра бліда з жовтуватим відтінком, суха, лущиться. В крові: еритроцити -1,8-1012/л, НЬ85 г/л, лейкоцити - 500-109 /л, формула: мієлобласти - 78%, нейтр.- 15%, лімф.- 7%. Який найбільш імовірний діагноз? An 18-year-old patient consulted a doctor due to pronounced weakness, fever, progressive weight loss, dizziness. Objectively: with a height of 165 cm, weight - 40 kg, the skin is pale with a yellowish tint, dry, scaly. In the blood: erythrocytes -1.8-1012/l, Hb85 g/l, leukocytes - 500-109 /l, formula: myeloblasts - 78%, neutr.- 15%, lymph - 7%. What is the most likely diagnosis?

Хронічний лімфобластний лейкоз Chronic lymphoblastic leukemia

Хронічна анемія Chronic anemia

Гострий мієлобластний лейкоз Acute myeloblastic leukemia

Гострий лімфобластний лейкоз Acute lymphoblastic leukemia

Лейкемоїдна реакція Leukemoid reaction

24 / 200
Хвора 42-х років госпіталізована на 12й день хвороби. Захворювання розпочалося гостро - остуда, що змінилася відчуттям жару, температура - 39,5oC, інтоксикація. Напад лихоманки продовжувався 6 годин, після чого температура знизилася до 36,5°C з надмірною пітливістю. Напади лихоманки повторювалися кожен 3-й день. Об’єктивно: на губах герпетичний висип, легка жовтяничність склер. Гепатолієнальний синдром. Аналіз крові: анемія, лейкопенія, лімфоцитоз, швидкість осідання еритроцитів -19 мм/год. Який найбільш інформативний метод діагностики при цьому захворюванні? A 42-year-old patient was hospitalized on the 12th day of the illness. The disease began acutely - a cold, which was replaced by a feeling of heat, temperature - 39.5oC, intoxication. The attack of fever continued 6 hours, after which the temperature dropped to 36.5°C with excessive sweating. Fever attacks recurred every 3rd day. Objectively: herpetic rash on the lips, mild sclera jaundice. Hepatolienal syndrome. Blood analysis: anemia, leukopenia, lymphocytosis , erythrocyte sedimentation rate -19 mm/h. What is the most informative method of diagnosis in this disease?

Бакпосів сечі Urine backup

Печінкові проби Liver tests

Посів крові на стерильність Blood culture for sterility

Мазок та товста крапля крові Smear and thick drop of blood

Реакція Відаля Vidal reaction

25 / 200
Хворий напередодні відчув легке нездужання, незначний головний біль, слабкість. Сьогодні температура підвищилася до 38,5oC з остудою, значним головним болем, переважно в лобовій ділянці, вираженим болем при рухах очних яблук. Шкіра обличчя і кон’юнктива гіперемовані. Виник сухий надсадний кашель. Зів гіпере-мований, на м’якому піднебінні зерниста енантема, місцями точкові крововиливи. Над легенями дихання з жорстким відтінком. Який найбільш імовірний діагноз? The day before, the patient felt a slight malaise, slight headache, weakness. Today the temperature rose to 38.5oC with a chill, significant headache, mainly in the frontal area, severe pain when moving the eyeballs. The skin of the face and the conjunctiva are hyperemic. A dry, percussive cough has appeared. The throat is hyperemic, on the soft palate there is a granular enanthema, in places point hemorrhages. Over the lungs, breathing with a hard shade. What is the most likely diagnosis?

Грип Flu

Ентеровірусна інфекція Enterovirus infection

Черевний тиф Typhoid

Гарячка Ку Ku fever

Висипний тиф Typhoid

26 / 200
Хвора 36-ти років скаржиться на задишку, відчуття стискання в правій половині грудної клітки, підвищення температури до 38,7oC, кашель з виділенням невеликої кількості слизово-гнійного харкотиння. Хворіє понад тиждень. Скарги пов’язує з переохолодженням. Об’єктивно: легкий акроціаноз губ, пульс ритмічний, 90/хв., артеріальний тиск - 140/85 мм рт.ст. Права половина грудної клітки відстає в акті дихання. Перкусія - справа нижче кута лопатки прослуховується тупість з межею до верху. В цій ділянці дихання відсутнє. Який найбільш імовірний діагноз? A 36-year-old patient complains of shortness of breath, a feeling of tightness in the right half of the chest, an increase in temperature to 38.7oC, a cough with the release of a small amount of mucous-purulent sputum . Has been sick for more than a week. Complaints are related to hypothermia. Objectively: slight acrocyanosis of the lips, rhythmic pulse, 90/min., blood pressure - 140/85 mm Hg. The right half of the chest lags behind in the act of breathing. Percussion - Dullness is heard on the right below the angle of the scapula with a border to the top. There is no breathing in this area. What is the most likely diagnosis?

Абсцес легені Lung abscess

Ателектаз легені Atelectasis of the lung

ТЕЛА BODIES

Ексудативний плеврит Exudative pleurisy

Позагоспітальна пневмонія Community-acquired pneumonia

27 / 200
До приймального покою швидкою допомогою доставлено хворого 46-ти років зі скаргами на різкий, нападоподібний біль у правій поперековій ділянці, що іррадіює у пахвинну ділянку та на внутрішню поверхню стегна. Біль з’явився раптово кілька годин тому. Напередодні у хворого з’явилася профузна безболісна гематурія зі згустками крові черв’якоподібної форми. Раніше нічим не хворів. Яке захворювання можна припустити в першу чергу? A 46-year-old patient was brought to the hospital by ambulance with complaints of sharp, attack-like pain in the right lumbar area, radiating to the inguinal area and to the inner surface of the thigh . The pain appeared suddenly a few hours ago. The day before, the patient had profuse painless hematuria with worm-shaped blood clots. He had never been sick before. What disease can be assumed first of all?

Некротичний папіліт Necrotic papillitis

Сечокам’яна хвороба, камінь правої нирки Urolithiasis, right kidney stone

Пухлина сечового міхура Bladder tumor

Рак правої нирки Right kidney cancer

Гострий гломерулонефрит Acute glomerulonephritis

28 / 200
Хворий 30-ти років, який страждав на головний біль, при піднятті важкого предмету відчув сильний головний біль, що нагадував удар по голові. З’явилися нудота, блювання, легке запаморочення. Об’єктивно через добу: виражений менінгеаль-ний синдром, температура тіла - 37,6°C. Лікар припустив субарахноїдальний крововилив. Яке додаткове обстеження необхідно провести в першу чергу, щоб підтвердити діагноз? A 30-year-old patient who suffered from headaches, when lifting a heavy object felt a severe headache, reminiscent of a blow to the head. Nausea, vomiting appeared , mild dizziness. Objectively, after a day: severe meningeal syndrome, body temperature - 37.6°C. The doctor assumed a subarachnoid hemorrhage. What additional examination should be conducted first of all to confirm the diagnosis?

Люмбальна пункція Lumbar puncture

Комп’ютерна томографія Computed tomography

Церебральна ангіографія Cerebral angiography

Рентгенографія черепа X-ray skull

Реоенцефалографія Rheoencephalography

29 / 200
Хвора 37-ми років скаржиться на постійні тупі болі в підребер’ях з іррадіацією в спину, які посилюються після їжі. Турбує здуття живота, часті випорожнення з домішками неперетравленої їжі. Хворіє понад 5 років, схудла на 15 кг. Об’єктивно: помірне здуття живота, болючість в зоні Шоффара, точках Дежардена, Мейо-Робсона. Який з методів дослідження буде найбільш інформативним для підтвердження зовнішньосекреторної недостатності підшлункової залози? A 37-year-old patient complains of constant dull pain in the hypochondrium with radiation to the back, which worsens after eating. She is bothered by bloating, frequent bowel movements with impurities of undigested food. She has been sick for more than 5 years, she has lost 15 kg. Objectively: moderate abdominal distension, tenderness in the Shofar zone, Desjardins, Mayo-Robson points. Which of the research methods will be the most informative to confirm exocrine pancreatic insufficiency?

УЗД органів черевної порожнини Ultrasound of abdominal organs

ЕГДС EGDS

Пероральна холецистографія Oral cholecystography

Ретроградна панкреатографія Retrograde pancreatography

Копрограма Coprogram

30 / 200
У хворого 59-ти років, який 15 років страждає на гіпертонічну хворобу, на тлі гіпертонічного кризу розвинулася гостра лі-вошлуночкова недостатність. Який препарат найдоцільніше призначити цьому хворому? A 59-year-old patient, who has been suffering from hypertension for 15 years, developed acute left ventricular failure against the background of a hypertensive crisis. What drug is the most appropriate to prescribe to this patient?'

Каптоприл Captopril

Ніфедипін Nifedipine

Нітропрусид натрію Sodium nitroprusside

Пропранолол Propranolol

Спіронолактон Spironolactone

31 / 200
Хворий 62-х років скаржиться на висип в ділянці лівої гомілки. Хворіє більше 2-х місяців. Лікувався самостійно, застосовуючи різні мазі та компреси, без покращення. Об’єктивно: на передній поверхні лівої гомілки вогнище хронічного запального характеру з чіткими краями. Поверхня нерівна, сосочкоподібні розростання. При натисканні виділяються краплі гною. Хворий відчуває незначний біль. RW, PIT, РІФ, реакція Манту - негативні. Яка найбільш імовірна патологія у даного хворого? A 62-year-old patient complains of a rash in the area of the left leg. He has been ill for more than 2 months. He was treated independently, using various ointments and compresses, without improvement. About objectively: on the front surface of the left lower leg, there is a focus of chronic inflammation with clear edges. The surface is uneven, papilla-like growths. When pressed, drops of pus are released. The patient feels slight pain. RW, PIT, RIF, Mantoux reaction are negative. What is the most likely pathology in this patient the patient?

Туберкульозний вовчак Tuberculous lupus

Бородавчастий туберкульоз Warty tuberculosis

Лейшманіоз Leishmaniasis

Третинний сифіліс Tertiary syphilis

Піодермія Pyoderma

32 / 200
Забійник 49-ти років, профстаж роботи 14 років, 75% робочого часу працює відбійним молотком, що генерує вібрацію 30-45 Гц. Скаржиться на болі і заніміння кистей, що непокоять у період відпочинку, болі у ліктьових і променевозап’ясткових суглобах, у шийному відділі хребта, поганий сон, періодичні ниючі болі у ділянці серця. Розвиток якого захворювання можна передбачити на основі скарг хворого? The killer is 49 years old, has 14 years of experience, works 75% of the time with a jackhammer that generates 30-45 Hz vibration. He complains of pain and numbness in his hands , disturbing during the rest period, pains in the elbow and wrist joints, in the cervical spine, poor sleep, periodic aching pains in the area of the heart. The development of which disease can be predicted based on the patient's complaints?

Остеохондроз шийного відділу хребта Osteochondrosis of the cervical spine

Неврит ліктьового нерва Ulnar nerve neuritis

Деформівний артроз ліктьових суглобів Deformative arthrosis of elbow joints

Вібраційна хвороба Vibration sickness

Синдром Рейно Raynaud's syndrome

33 / 200
У доярки на правій кисті наявна виразка - 3х3 см, вкрита чорним щільним струпом. Кисть різко набрякла, безболісна. Колір шкіри у ділянці набряку не змінений. Збільшений пахвовий лімфовузол справа. Пальпація лімфовузла безболісна. Симптоми інтоксикації виражені помірно. Який найбільш імовірний діагноз? The milkmaid has an ulcer on her right hand - 3x3 cm, covered with a black dense scab. The hand is sharply swollen, painless. The color of the skin in the area of swelling has not changed. An enlarged axillary lymph node on the right. Palpation of the lymph node is painless. Symptoms of intoxication are moderately pronounced. What is the most likely diagnosis?

Шкірно-бубонна форма чуми Cutaneous-bubonic form of plague

Банальний карбункул Tranical carbuncle

Бешиха Beshikha

Виразково-бубонна форма туляремії Ulcer-bubonic form of tularemia

Шкірна форма сибірки Cutaneous form of anthrax

34 / 200
Хвора 63-х років оперована з приводу багатовузлового еутиреоїдного зобу великих розмірів. З технічними труднощами вимушено виконана субтотальна резекція обох часток щитоподібної залози. На 4-й день після операції з’явилися судоми м’язів обличчя і верхніх кінцівок, біль в животі. Позитивні симптоми Хвостека і Труссо. Чим найімовірніше зумовлений такий стан хворої? A 63-year-old patient was operated on for a large multinodular euthyroid goiter. With technical difficulties, a subtotal resection of both lobes of the thyroid gland was forced to be performed. On the 4th day after the operation muscle spasms of the face and upper limbs appeared, pain in the abdomen. Positive symptoms of Khvostek and Trousseau. What is the most likely cause of this condition of the patient?

Післяопераційний гіпотиреоз Postoperative hypothyroidism

Пошкодження зворотнього нерву Injury of the recurrent nerve

Недостатність паращиподібних залоз Insufficiency of parasympathetic glands

Трахеомаляція Tracheomalacia

Тиреотоксичний криз Thyrotoxic crisis

35 / 200
-х років скаржиться на підвищення температури тіла до 37,4o 35. C, появу геморагічного висипу на нижніх кінцівках, біль в попереку, червоний колір сечі. Захворіла 3 дні тому після переохолодження. Об’єктивно: шкіра бліда, на поверхні гомілок і стегон - дрібний геморагічний симетричний висип. Частота серцевих скорочень - 90/хв., артеріальний тиск -115/90 мм рт.ст. Симптом Пастернацького слабкопо-зитивний з обох боків. В крові: лейкоцити - 9,6*109 /л, тромбоцити - 180*109 /л, швидкість осідання еритроцитів - 31 мм/год. В сечі: білок - 0,33 г/л, ер. Змінені - 3-40 в п/з, лейкоцити - 5-8 в п/з. Який з наведених препаратів є патогенетично обґрунтованим для лікування в даному випадку? years old complains of an increase in body temperature to 37.4o 35. C, the appearance of a hemorrhagic rash on the lower extremities, pain in the lower back, red color of urine. She became ill 3 days ago after hypothermia. Objectively: the skin is pale, on the surface of the lower legs and thighs - a small hemorrhagic symmetrical rash. Heart rate - 90/min., blood pressure -115/90 mm Hg. Pasternacki's symptom is weakly positive in both sides. In blood: leukocytes - 9.6*109 /l, platelets - 180*109 /l, erythrocyte sedimentation rate - 31 mm/h. In urine: protein - 0.33 g/l, er. Changed - 3- 40 in p/z, leukocytes - 5-8 in p/z. Which of the following drugs is pathogenetically justified for treatment in this case?

Гепарин Heparin

Аскорбінова кислота Ascorbic acid

Рутин Routine

Вікасол Vikasol

Кальцію глюконат Calcium gluconate

36 / 200
Жінка 36-ти років скаржиться на біль, обмеження рухів у дрібних суглобах рук, утруднене ковтання твердої їжі, слабкість, сухий кашель. Об’єктивно: шкіра кистей та передпліччя щільна, гладенька. Проксимальні суглоби II-IV пальців кистей рук набряклі, болючі при пальпації. Над легенями сухі розсіяні хрипи, межі серця зміщені вліво на 2 см, тони приглушені. У крові: швидкість осідання еритроцитів - 36 мм/год, 7-глобуліни - 24%. У сечі: змін немає. Який найбільш імовірний діагноз? A 36-year-old woman complains of pain, limitation of movement in the small joints of the hands, difficulty swallowing solid food, weakness, dry cough. Objectively: the skin of the hands and the forearm is dense, smooth. Proximal joints II-IV of the fingers of the hands are swollen, painful on palpation. Over the lungs, dry scattered rales, the borders of the heart are shifted to the left by 2 cm, tones are muffled. In the blood: erythrocyte sedimentation rate - 36 mm/h, 7- globulins - 24%. In the urine: there are no changes. What is the most likely diagnosis?

Системна склеродермія Systemic scleroderma

Ревматоїдний артрит Rheumatoid arthritis

Саркоїдоз Sarcoid

Дерматоміозит Dermatomyositis

Системний червоний вовчак Systemic lupus erythematosus

37 / 200
Жінка 49-ти років скаржиться на біль у плечових суглобах, що посилюється під час рухів, обмеження рухливості, короткочасну вранішню скутість. Хворіє декілька років. В минулому займалася спортивною гімнастикою. На рентгенограмі плечових суглобів - звуження суглобової щілини, субхондральний остеосклероз, остеофіти у нижньовнутрішньому відділі головки плечової кістки. Що лежить в основі патогенезу ураження суглобів у хворої? A 49-year-old woman complains of pain in the shoulder joints, which worsens during movements, limitation of mobility, short-term morning stiffness. She has been sick for several years. In the past, she was engaged in sports gymnastics. On the x-ray of the shoulder joints - narrowing of the joint space, subchondral osteosclerosis, osteophytes in the lower inner part of the head of the humerus. What is the basis of the pathogenesis of joint damage in the patient?

Порушення метаболізму хряща Cartilage metabolism disorder

Відкладення у синовіальній оболонці імунних комплексів Deposition of immune complexes in the synovial membrane

Пошкодження синовіальної оболонки інфекційним агентом Damage of the synovial membrane by an infectious agent

Відкладення у суглобах пірофосфату кальцію Calcium pyrophosphate deposits in the joints

Порушення обміну сечової кислоти Uric acid metabolism disorder

38 / 200
Жінка 40-ка років госпіталізована до клініки зі скаргами на задуху, яка триває кілька годин і не проходить від дії раніш ефективного астмопенту. З’явилися серцебиття та неспокій. 8 років хворіє на хронічний бронхіт. Об’єктивно: стан тяжкий, хвора сидить, спираючись руками на край столу, блідий ціаноз, на відстані чутно свистяче дихання. В легенях на тлі послабленого дихання вислуховуються в невеликій кількості розсіяні сухі хрипи. Пульс -108/хв., артеріальний тиск -140/80 мм рт.ст. Застосування яких препаратів у першу чергу є обов’язковим у лікуванні хворої? A 40-year-old woman was admitted to the clinic with complaints of shortness of breath, which lasts for several hours and does not go away with the previously effective astmopent. Palpitations and restlessness appeared. She has been suffering from chronic bronchitis for 8 years. Objectively: the condition is severe, the patient is sitting, resting her hands on the edge of the table, pale cyanosis, whistling breathing is heard in the distance. A small number of scattered dry rales are heard in the lungs against the background of weakened breathing. Pulse -108/ min., blood pressure - 140/80 mm Hg. Which drugs should be used primarily in the treatment of the patient?

Глюкокортикоїди Glucocorticoids

Стабілізатори мембран опасистих клітин Stabilizers of mast cell membranes

Холінолітики Cholinolytics

Антигістамінні Antihistamines

Адреноміметики Adrenomimetics

39 / 200
Жінка 58-ми років скаржиться на безпричинну появу синців, слабкість, кровоточивість ясен, запаморочення. Об’єктивно: слизові оболонки та шкірні покриви бліді, з численними крововиливами різної давнини. Лімфатичні вузли не збільшені. Пульс -100/хв., артеріальний тиск -110/70 мм рт.ст. З боку внутрішніх органів змін не виявлено. У крові: еритроцити - 3,01*1012/л, НЬ- 92 г/л, КП- 0,9, анізоцитоз, пойкілоцитоз, лейкоцити - 10-109 /л, е.- 2%, п.- 12%, с.- 68%, л.-11%, м.- 7%, швидкість осідання еритроцитів - 12 мм/год. Додаткове визначення якого лабораторного показника найбільш доцільне для встановлення діагнозу? A 58-year-old woman complains of unexplained bruising, weakness, bleeding gums, dizziness. Objectively: mucous membranes and skin are pale, with numerous hemorrhages of various of ancient times. Lymph nodes are not enlarged. Pulse - 100/min., blood pressure - 110/70 mm Hg. No changes were detected on the part of internal organs. In blood: erythrocytes - 3.01*1012/l, Hb - 92 g /l, KP- 0.9, anisocytosis, poikilocytosis, leukocytes - 10-109 /l, e.- 2%, p.- 12%, s.- 68%, l.-11%, m.- 7% , the sedimentation rate of erythrocytes is 12 mm/hour. Additional determination of which laboratory indicator is the most appropriate for establishing a diagnosis?

Час згортання крові Blood clotting time

Осмотична резистентність еритроцитів Osmotic resistance of erythrocytes

Тромбоцити Platelets

Ретикулоцити Reticulocytes

Фібриноген Fibrinogen

40 / 200
Жінка 34-х років скаржиться на слабкість, зниження ваги на 12 кг за півроку, пітливість, серцебиття, дратівливість. Об’єктивно: щитоподібна залоза III ступеню, еластична, на тлі дифузного збільшення в правій частці вузол. Шийні лімфатичні вузли не збільшені. Яка лікувальна тактика найбільш обґрунтована? A 34-year-old woman complains of weakness, weight loss of 12 kg in six months, sweating, palpitations, irritability. Objectively: thyroid gland III degree, elastic , on the background of a diffuse increase in the right lobe of the nodes. The cervical lymph nodes are not enlarged. What treatment tactics are most justified?

Негайне хірургічне втручання Immediate surgery

Негайна телегамматерапія Immediate telegamma therapy

Призначення радіоактивного йоду Purpose of radioactive iodine

Операція після антитиреоїдної терапії Surgery after antithyroid therapy

Консервативна антитиреоїдна терапія Conservative antithyroid therapy

41 / 200
Хворому 25-ти років поставлено діагноз: перелом кісток тазу. Стан тяжкий, пульс - 116/хв., артеріальний тиск - 100/60 мм рт.ст. Живіт напружений, болючий, симптоми подразнення очеревини слабко виражені. В крові: еритроцити - 3,2*1012/л; нЬ- 110 г/л; Ht0,37; лейкоцити - 10*109 /л; паличкоядерні -10%. Який додатковий метод обстеження найбільш інформативний? A 25-year-old patient was diagnosed with a fracture of the pelvis. The condition is serious, pulse - 116/min, blood pressure - 100/60 mm Hg. The abdomen is tense, painful, the symptoms of peritoneal irritation are weakly expressed. In the blood: erythrocytes - 3.2*1012/l; Hb - 110 g/l; Ht0.37; leukocytes - 10*109 /l; rod cells - 10%. What additional the examination method is the most informative?

Лапароцентез Laparocentesis

Лапароскопія Laparoscopy

Ультразвукове дослідження черева Abdominal ultrasound examination

Оглядова рентгенограма черева Abdominal radiograph

Фіброгастродуоденоскопія Fibrogastroduodenoscopy

42 / 200
7-ми років в анамнезі відзначається пікова температура до 40o 42. C, протягом 3-х місяців спостерігається веретеноподібна припухлість суглобів пальців рук, колінного і гомілковостопного суглобів, біль у верхній частині грудини, шийному відділі хребта. Який найбільш імовірний діагноз? For 7 years, the anamnesis has a peak temperature up to 40o 42.C, for 3 months there has been a spindle-shaped swelling of the joints of the fingers, knee and ankle joints, pain in the upper sternum, cervical spine. What is the most likely diagnosis?

Токсичний синовіїт Toxic synovitis

Ювенільний ревматоїдний артрит Juvenile rheumatoid arthritis

Остеоартрит Osteoarthritis

Септичний артрит Septic arthritis

Іостра ревматична лихоманка Acute rheumatic fever

43 / 200
Внаслідок дорожньо-транспортної пригоди потерпілий отримав множинні переломи кінцівок та кісток тазу. В анамнезі: гемофілія А. Під час обстеження формуються гематоми на ушкоджених ділянках. Стан погіршується. Артеріальний тиск -90/50 мм рт.ст. Вкажіть найбільш доцільну комбінацію інфузійних засобів для лікування пацієнта після застосування поліглюкіну та сольових розчинів: As a result of a traffic accident, the victim received multiple fractures of the limbs and pelvic bones. History: hemophilia A. During the examination, hematomas form on the damaged areas. The condition worsens. Arterial pressure -90/50 mm Hg Specify the most appropriate combination of infusion agents for the treatment of the patient after the use of polyglucin and saline solutions:

Еритроцитарна маса Erythrocyte mass

Кріопреципітат, еритроцитарна маса Cryoprecipitate, erythrocyte mass

Кріопреципітат, глюкоза Cryoprecipitate, glucose

Свіжозаморожена плазма, альбумін Fresh-frozen plasma, albumin

Еритроцитарна маса, свіжозаморожена плазма Erythrocyte mass, fresh frozen plasma

44 / 200
Хворий 70-ти років скаржиться на слабкість, запаморочення, короткочасні періоди непритомності, біль в ділянці серця. Під час аускультації серця: частота серцевих скорочень - 40/хв., тони ритмічні, I тон періодично значно посилений, артеріальний тиск -160/90 мм рт.ст. Яка найбільш імовірна причина гемодинамічних порушень? A 70-year-old patient complains of weakness, dizziness, short-term periods of unconsciousness, pain in the heart area. During auscultation of the heart: heart rate - 40/min. , tones are rhythmic, I tone is periodically significantly increased, blood pressure -160/90 mm Hg. What is the most likely cause of hemodynamic disturbances?

АВ блокада I ступеня AV blockade of the 1st degree

Повна блокада лівої ніжки пучка Гіса Complete blockade of the left leg of the bundle of His

АВ блокада III ступеня AV block III degree

Синусова брадикардія Sinus bradycardia

Брадисистолічна форма миготливої аритмії Bradysystolic form of atrial fibrillation

45 / 200
У дитини 6-ти місяців 8 годин тому раптово виник інтенсивний переймоподібний біль в животі. Періоди неспокою дитини тривають 5 хвилин, з проміжком в 15 хвилин. Випорожнень не було. Годину тому виникла прямокишкова кровотеча. Під час пальпації: живіт м’який, болючий у правій половині, де нечітко визначається видовжене утворення - 10х4 см. При ректальному дослідженні - сліди крові. Який найбільш імовірний діагноз? 8 hours ago, a 6-month-old child suddenly had intense spasm-like abdominal pain. The child's periods of restlessness last 5 minutes, with an interval of 15 minutes. There were no bowel movements . Rectal bleeding occurred an hour ago. During palpation: the abdomen is soft, painful in the right half, where an elongated formation - 10x4 cm is vaguely defined. During rectal examination - traces of blood. What is the most likely diagnosis?

Кишкова кровотеча Intestinal bleeding

Гострий ентероколіт Acute enterocolitis

Ілеоцекальна інвагінація Ileocecal intussusception

Заворот кишківника Intestinal volvulus

Гострий апендицит Acute appendicitis

46 / 200
У хворого 35-ти років, що страждає на хронічний гломерулонефрит і останні 3 роки перебуває на гемодіалізі, з’явилися перебої в роботі серця, гіпотонія, зростаюча слабкість, задишка. На ЕКГ: брадикардія, атріовентрикулярна блокада I ст., високі загострені зубці Т. Напередодні - грубе порушення питного та дієтичного режимів. Яка найбільш імовірна причина вказаних змін? A 35-year-old patient who suffers from chronic glomerulonephritis and has been on hemodialysis for the past 3 years has had heart failure, hypotension, increasing weakness, shortness of breath. On the ECG: bradycardia, atrioventricular block of the 1st degree, high pointed T waves. The day before - a gross violation of drinking and dietary regimes. What is the most likely cause of the indicated changes?

Гіпергідратація Hyperhydration

Гіпернатріємія Hypernatremia

Гіперкаліємія Hyperkalemia

Гіпокальціємія Hypocalcemia

Гіпокаліємія Hypokalemia

47 / 200
на раптовий початок хвороби: лихоманка до 39,0o 47. C, неправильного типу, з гострим болем у грудях. Харкотиння з гнилісним запахом та домішками крові, до 400 мл за добу. В ході перкусії: над ураженою ділянкою вкорочення перкуторного звуку, посилене голосове тремтіння. У харкотинні був виділений анаеробний стрептокок. Яку хворобу можна припустити в першу чергу? for the sudden onset of the disease: fever up to 39.0o 47. C, of the wrong type, with sharp pain in the chest. Sputum with a putrid smell and impurities of blood, up to 400 ml per day. During percussion: shortening of the percussion sound over the affected area, increased vocal tremor. Anaerobic streptococcus was isolated in the sputum. What disease can be assumed first of all?

Абсцес легені Lung abscess

Туберкульоз Tuberculosis

Абсцедивна пневмонія Abscess pneumonia

Бронхоектатична хвороба Bronchoectatic disease

Гангрена легені Gangrene of the lungs

48 / 200
Хворий 46-ти років скаржиться на свербіння шкіри, пітливість, особливо вночі, підвищення температури до 38,6oC. Об’єктивно: на шкірі грудей сліди розчухів, надключичні лімфовузли збільшені до 3 см, зі шкірою не спаяні. Який метод дослідження є найбільш доцільним? A 46-year-old patient complains of itching of the skin, sweating, especially at night, an increase in temperature up to 38.6oC. Objectively: the skin of the chest has traces of scratching, supraclavicular the lymph nodes are enlarged up to 3 cm, they are not connected to the skin. What research method is the most appropriate?

Оглядова рентгенографія грудної порожнини Comprehensive radiography of the chest cavity

Загальний аналіз крові General blood test

Імунограма Immunogram

Білки крові та білкові фракції Blood proteins and protein fractions

Біопсія збільшеного лімфовузла Biopsy of an enlarged lymph node

49 / 200
У хворого 50-ти років раптово виникли кинджальний біль у потиличній ділянці та блювання. Об’єктивно: сопор, гіперемія шкіри обличчя, артеріальний тиск - 210/120 мм рт.ст., пульс - 60/хв., напружений, температура тіла - 37,8°C. Горизонтальний ністагм. Виражені рефлекси орального автоматизму. Сухожилкові рефлекси рівномірні. Ригідність потиличних м’язів, двобічний симптом Керніга. Який попередній діагноз? A 50-year-old patient suddenly developed a stabbing pain in the back of the head and vomiting. Objectively: sopor, hyperemia of the skin of the face, blood pressure - 210/120 mm Hg, pulse - 60/min, strained, body temperature - 37.8°C. Horizontal nystagmus. Pronounced reflexes of oral automatism. Tendon reflexes are even. Rigidity of occipital muscles, bilateral Kernig sign. What is the previous diagnosis?

Субарахноїдальний крововилив Subarachnoid hemorrhage

Менінгококовий менінгіт Meningococcal meningitis

Гостра гіпертонічна енцефалопатія Acute hypertensive encephalopathy

Субдуральна гематома Subdural hematoma

Геморагічний паренхіматозний інсульт Hemorrhagic parenchymal stroke

50 / 200
Хворий 31-го року звернувся до лікаря зі скаргами на збільшення лімфовузлів зліва над ключицею. Біль у грудях та животі, свербіння шкіри вночі. Хворіє протягом року. Об’єктивно: пальпуються збільшені неболючі лімфовузли зліва в надключичній ділянці. Печінка і селезінка не збільшені. В крові: Hb- 70 г/л, лейкоцити - 19,6*109 /л, еоз.- 1%, пал.- 8%, с.-83%, лімф.- 2%, мон.- 6%, швидкість осідання еритроцитів - 55 мм/год, тромбоцити - 58*109 /л. На Rо-графiї грудної порожнини у верхівці правої легені визначається інфільтрат, який контрастує з легеневою тканиною. Які дослідження найбільш доцільно провести для підтвердження діагнозу? A 31-year-old patient turned to the doctor with complaints of an increase in lymph nodes on the left above the collarbone. Pain in the chest and abdomen, itching of the skin at night. He has been sick for a year. About' objectively: enlarged painless lymph nodes are palpated on the left in the supraclavicular area. The liver and spleen are not enlarged. In the blood: Hb - 70 g/l, leukocytes - 19.6*109 /l, eos. - 1%, pal. - 8%, s .-83%, lymph - 2%, mon. - 6%, erythrocyte sedimentation rate - 55 mm/h, platelets - 58*109 /l. On Ro-graphy of the chest cavity in the apex of the right lung, an infiltrate is determined, which contrasts with lung tissue. What studies are most appropriate to conduct to confirm the diagnosis?

Рентгенологічне дослідження шлунку X-ray examination of the stomach

Аналіз сечі на білок Бен-Джонса Urine analysis for Ben-Jones protein

Біопсія лімфовузла Lymph node biopsy

Трепанобіопсія Trepanobiopsy

Стернальна пункція Sternal puncture

51 / 200
Хворий 46-ти років вперше помітив набряки на ногах, слабкість, відчуття ’’повноти” та важкості в правому підребер’ї. 20 років хворіє на ревматоїдний артрит. Збільшені печінка і селезінка, щільної консистенції. Креатинін крові - 0,23 ммоль/л, протеїнемія - 68 г/л, холестерин - 4,2 ммоль/л, питома вага сечі - 1012, протеїнурія - 3,3 г/л, поодинокі воскоподібні циліндри, еритроцити вилужені в п/з, лейкоцити- 5-6 в п/з. Яке ускладнення найбільш імовірне? A 46-year-old patient first noticed swelling in his legs, weakness, a feeling of 'fullness' and heaviness in the right hypochondrium. He has been suffering from rheumatoid arthritis for 20 years. Enlarged liver and spleen, dense consistency. Blood creatinine - 0.23 mmol/l, proteinemia - 68 g/l, cholesterol - 4.2 mmol/l, specific gravity of urine - 1012, proteinuria - 3.3 g/l, single wax-like cylinders, erythrocytes leached out in p/z, leukocytes - 5-6 in p/z. What is the most likely complication?

Амілоїдоз нирок Kidney amyloidosis

Гострий гломерулонефрит Acute glomerulonephritis

Серцева недостатність Heart failure

Хронічний пієлонефрит Chronic pyelonephritis

Хронічний гломерулонефрит Chronic glomerulonephritis

52 / 200
У дівчинки 12-ти років хронічний вірусний гепатит С впродовж 7-ми років. Стан погіршився за останні 6 місяців. Схудла, з’явилася жовтяничність шкірних покривів і склер. Виражене свербіння шкіри, судинні ’зірочки”’ носові кровотечі. Асцит. Печінка +4 см, селезінка на рівні пупкової лінії. З розвитком якого синдрому пов’язане виражене свербіння шкіри у хворої? A 12-year-old girl has chronic viral hepatitis C for 7 years. The condition has worsened over the past 6 months. She has lost weight, jaundice of the skin and sclera has appeared Pronounced itching of the skin, vascular 'stars' nosebleeds. Ascites. Liver +4 cm, spleen at the level of the umbilical line. The development of which syndrome is associated with the development of pronounced itching of the skin in the patient?

Гіперспленізм Hypersplenism

Гепатоцелюлярна недостатність Hepatocellular failure

Холестаз Cholestasis

Портальна гіпертензія Portal hypertension

Набряково-асцитичний синдром Oedema-ascites syndrome

53 / 200
Спортсмен 26-ти років впав з упором на правий плечовий суглоб. Зазначає різкий біль, обмеження пасивних рухів, активні рухи в плечовому суглобі неможливі. Відведену і зігнуту у плечовому суглобі травмовану руку підтримує здоровою. Об’єктивно: в ділянці дельтоподібного м’яза западіння, під шкірою різко випинається акроміальний відросток лопатки, голівка плечової кістки пальпується під дзьобоподібним відростком. Який найбільш імовірний клінічний діагноз? A 26-year-old athlete fell with an emphasis on the right shoulder joint. He notes sharp pain, limitation of passive movements, active movements in the shoulder joint are impossible. Abducted and bent shoulder the joint keeps the injured hand healthy. Objectively: there is a depression in the area of the deltoid muscle, the acromial process of the scapula protrudes sharply under the skin, the head of the humerus can be palpated under the beak-like process. What is the most likely clinical diagnosis?

Перелом голівки плечової кістки Fracture of humeral head

Вивих плеча Shoulder dislocation

Перелом хірургічної шийки плеча Fracture of the surgical neck of the shoulder

Відрив великого пагорбка плечової кістки Detachment of the greater tubercle of the humerus

Перелом анатомічної шийки плеча Fracture of the anatomical neck of the humerus

54 / 200
У хворого 33-х років через 3 місяці після тиреоїдектомії у зв’язку з дифузним токсичним зобом з’явилися остуда, сонливість, апатія, зниження апетиту, закрепи, змінився голос. Об’єктивно: шкіра суха, холодна, бліда, обличчя одутле, щільний набряк губ, кінцівок. Щитоподібна залоза не пальпується. Пульс - 52/хв., артеріальний тиск -100/60 мм рт.ст. Тони серця послаблені. Добова протеінурія -100 мг Стан хворого зумовлений: A 33-year-old patient developed chills, drowsiness, apathy, decreased appetite, constipation, 3 months after thyroidectomy due to diffuse toxic goiter. the voice has changed. Objectively: the skin is dry, cold, pale, the face is swollen, dense swelling of the lips, limbs. The thyroid gland is not palpable. Pulse - 52/min., blood pressure - 100/60 mm Hg. Heart tones are weakened. Daily proteinuria - 100 mg. The patient's condition is due to:

Серцевою недостатністю With heart failure

Рецидивом токсичного зобу Relapse of toxic goiter

Нефротичним синдромом Nephrotic syndrome

Розвитком гіпотиреозу Development of hypothyroidism

Розвитком вузлового токсичного зобу Development of nodular toxic goiter

55 / 200
У чоловіка 37-ти років після гострої респіраторної інфекції поступово протягом тижня розвинулася слабкість ніг, а згодом м’язів тулуба, рук, шиї, обличчя, утруднення ковтання, мови. В неврологічному статусі: дисфагія, дизартрія, дисфонія, порушення дихання, діяльності серця, тетрапарез. Найбільш імовірно, що хворого треба направити: A 37-year-old man, after an acute respiratory infection, gradually developed weakness in the legs over the course of a week, and later in the muscles of the trunk, arms, neck, face, difficulty swallowing, language. In the neurological status: dysphagia, dysarthria, dysphonia, breathing disorders, cardiac activity, tetraparesis. It is most likely that the patient should be referred to:

На амбулаторне лікування For outpatient treatment

До терапевтичного відділення To the therapeutic department

До інфекційного відділення To the infectious department

До неврологічного відділення To the neurology department

До реанімаційного відділення To the intensive care unit

56 / 200
У дівчинки 7-ми років скарги на слабкість, підвищену втомлюваність, підвищення температури тіла до 38°С, малу кількість виділеної за добу сечі, кольору ’’м’ясних помиїв”’ Об’єктивно: блідість шкіри, набряк обличчя, рук, ніг, біль у попереку. У крові: еритроцити - 2,7*1012/л, Hb- 90 г/л, лейкоцити - 17*109 /л, е.- 10%, п.- 4%, с.-60%, л.- 16%, м.- 10%, швидкість осідання еритроцитів - 30 мм/год. В сечі: лейкоц.-15 в п/з, еритр.- 30 в п/з, гіалінові циліндри -8-10 в п/з, білок - 4 г/л. Холестерин в крові - 8 ммоль/л, загальний білок - 43 г/л. Який провідний механізм розвитку набряків? A 7-year-old girl complains of weakness, increased fatigue, an increase in body temperature up to 38°С, a small amount of urine released per day, color ``m'' of clear scum'' Objectively: pallor of the skin, swelling of the face, hands, feet, lower back pain. In the blood: erythrocytes - 2.7*1012/l, Hb - 90 g/l, leukocytes - 17*109 /l, e.- 10%, p.- 4%, p.-60%, l.- 16%, m.- 10%, erythrocyte sedimentation rate - 30 mm/h. In urine: leukocyte - 15 in p/z , erythr. - 30 in p/z, hyaline cylinders -8-10 in p/z, protein - 4 g/l. Blood cholesterol - 8 mmol/l, total protein - 43 g/l. What is the leading mechanism of edema development ?

Дизелектролітні порушення Dielectrolyte violations

Зниження осмотичного тиску крові Decrease in blood osmotic pressure

Зниження онкотичного тиску крові Reduction of blood oncotic pressure

Порушення серцевої діяльності Cardiac disorders

Гіперальдостеронізм Hyperaldosteronism

57 / 200
Пацієнт 38-ми років висловлює скарги на сильний пекучий біль у ділянці кисті та передпліччя. В анамнезі: різана рана во-лярної поверхні нижньої третини передпліччя. Пошкодження якого нерва найчастіше викликає подібні болі? A 38-year-old patient complains of severe burning pain in the area of the hand and forearm. History: cut wound of the volar surface of the lower third of the forearm. Damage to which nerve most often causes such pains?

Променевого Radian

Ліктьового Elbow

Серединного Middle

Міжкісткового Interosseous

Пахвового Axillary

58 / 200
Хвора 35-ти років скаржиться на біль в правій гомілці, який посилюється під час ходьби. Об’єктивно: на правій гомілці за ходом варикозно розширеної малої підшкірної вени шкіра червона, гаряча на дотик, під час пальпації в верхній третині гомілки визначається довгасте болюче ущільнення розмірами 3х2 см. Яке ускладнення варикозної хвороби виникло у пацієнтки? A 35-year-old patient complains of pain in her right lower leg, which intensifies while walking. Objectively: on the right lower leg, along the course of a varicose small saphenous vein, the skin red, hot to the touch, during palpation in the upper third of the lower leg, an oblong painful induration measuring 3x2 cm is determined. What complication of varicose veins did the patient experience?

Фурункул Furnish

Лімфангіт Lymphangitis

Лімфаденіт Lymphadenitis

Тромбофлебіт Thrombophlebitis

Тромбоз глибоких вен Deep vein thrombosis

59 / 200
Мати привела на прийом до лікаря хлопчика 5-ти років зі скаргою на висип на обличчі, що з’явився 5 днів тому назад. Об’єктивно: на обличчі помітні декілька пухирів розмірами 0,5-1,5 см в діаметрі, з тонкими в’ялими кришками, виповнені серозним каламутним ексудатом і розташовані на гіперемованій, набряклій основі. Поряд з цими вогнищами ураження спостерігаються ерозії з виділенням серозно-гнійної рідини та товсті медовожовті кірки. Поставте клінічний діагноз: The mother brought a 5-year-old boy to the doctor with a complaint of a rash on his face that appeared 5 days ago. Objectively: on his face several blisters measuring 0.5-1.5 cm in diameter, with thin flaccid lids, filled with serous cloudy exudate and located on a hyperemic, swollen base are visible. Along with these lesions, erosions with the release of serous-purulent fluid and thick honey-yellow crusts. Make a clinical diagnosis:

Кандидоз Candida

Вульгарне імпетиго Vulgar impetigo

Дитяча екзема Children's eczema

Контактний дерматит Contact dermatitis

Простий пухирцевий лишай Liquor simplex

60 / 200
У 5-місячної дитини тричі рецидивував бронхообструкутивний синдром без попередніх катаральних явищ, практично постійно спостерігався кашель з важковід-ділюваним харкотинням. Випорожнення жирні, смердючі з перших днів життя. Припускається муковісцидоз. Яке дослідження може підтвердити діагноз? A 5-month-old child had three relapses of broncho-obstructive syndrome without previous catarrhal phenomena, cough with difficult sputum was observed almost constantly. Faeces were greasy, smelly from the first days of life. Cystic fibrosis is suspected. What research can confirm the diagnosis?

Копроцитограма Coprocytogram

Визначення рівня ліпідів у крові Determining the level of lipids in the blood

Рентгенографія органів грудної клітки X-ray of chest organs

Хлориди поту Sweat Chlorides

Сцинтиграфія легень Lung scintigraphy

61 / 200
Робітник працює у ливарні з кольоровими металами та сплавами протягом 12-ти років. У повітрі робочої зони зареєстровано підвищений вміст важких металів, оксидів вуглецю та азоту. Під час регулярного медичного огляду у хворого виявлено астено-вегетативний синдром, різкий біль в животі, закрепи, біль в ділянці печінки. У сечі - амінолевулінова кислота та копро-порфірін. В крові - ретикулоцитоз, зниження рівню гемоглобіну. Причиною такої інтоксикації є: A worker has been working in a foundry with non-ferrous metals and alloys for 12 years. In the air of the working area, an increased content of heavy metals, carbon and nitrogen oxides was registered. During regular medical examination of the patient revealed astheno-vegetative syndrome, sharp pain in the abdomen, constipation, pain in the liver area. In the urine - aminolevulinic acid and copro-porphyrin. In the blood - reticulocytosis, a decrease in the level of hemoglobin. The cause of such intoxication is:

Оксид азоту Nitrogen oxide

Цинк Zinc

Оксид вуглецю Carbon monoxide

Олово Tin

Свинець та його солі Lead and its salts

62 / 200
У хворого, який вживав три доби тому консервовані гриби - опеньки, спостерігаються розлад зору (диплопія, мідріаз), розлад мовлення, порушення ковтання. Яке харчове отруєння має місцє? A patient who consumed canned mushrooms - boletus mushrooms three days ago has vision disorders (diplopia, mydriasis), speech disorders, and swallowing disorders. What kind of food poisoning is there ?

Харчова токсикоінфекція Food poisoning

Ботулізм Botulism

Отруєння опеньками Poisoning by mushrooms

Отруєння солями свинцю Poisoning with lead salts

Фузаріотоксикоз Fusariotoxicosis

63 / 200
24-х років на 5-ту добу раптово підвищилася температура тіла до 38,7o 63. C. Скаржиться на слабкість, головний біль, біль внизу живота, роздратованість. Об’єктивно: артеріальний тиск - 120/70 мм рт.ст., пульс - 92/хв., температура тіла -38,7oC. Бімануально: матка збільшена до 12 тижнів вагітності, щільна, дещо болюча при пальпації, цервікальний канал пропускає 2 п/п, виділення помірні, каламутні, з неприємним запахом. У крові: лейкоцитоз зі зсувом вліво, лімфопенія, швидкість осідання еритроцитів - 30 мм/год. Який найбільш імовірний діагноз? 24 years old, on the 5th day, the body temperature suddenly rose to 38.7o 63. C. Complains of weakness, headache, pain in the lower abdomen, irritability . Objectively: blood pressure - 120/70 mm Hg, pulse - 92/min, body temperature -38.7oC. Bimanually: the uterus is enlarged up to 12 weeks of pregnancy, dense, somewhat painful on palpation, the cervical canal passes 2 p/p, secretions are moderate, cloudy, with an unpleasant smell. In the blood: leukocytosis with a shift to the left, lymphopenia, erythrocyte sedimentation rate - 30 mm/h. What is the most likely diagnosis?

Параметрит Parameter

Ендометрит Endometritis

Пельвіоперитоніт Pelvioperitonitis

Метрофлебіт Metrophlebitis

Лохіометра Lochiometer

64 / 200
З дому надійшла дитина 3-х років, у якої на тлі гіпертермічного синдрому, спричиненого грипом, протягом 50-ти хвилин тривають тоніко-клонічні генералізовані судоми. Терапія не проводилася. Для негайного лікування судомного синдрому необхідно: A 3-year-old child came from home, who has been having tonic-clonic generalized convulsions for 50 minutes against the background of hyperthermic syndrome caused by influenza. Therapy is not For the immediate treatment of a convulsive syndrome, it is necessary:

Ввести кальцій Enter calcium

Фенобарбітал Phenobarbital

Ввести діазепам Enter diazepam

Жарознижувальні Antipyretics

Фізичні методи охолодження Physical methods of cooling

65 / 200
Для визначення впливу мікроклімату на функціональний стан організму досліджувалися наступні фізіологічні показники напруження функцій органів і систем, які беруть участь в теплообмінних процесах: пульс, артеріальний тиск, кількість дихальних актів за хвилину, температура шкіри, потовиділення за хвилину, швидкість зорової та слухової реакцій. Назвіть показник, що найбільш об’єктивно відображає напруження процесів терморегуляцій організму: To determine the influence of the microclimate on the functional state of the body, the following physiological indicators of the stress on the functions of organs and systems involved in heat exchange processes were studied: pulse, blood pressure, number of respiratory acts per minute, skin temperature, sweating per minute, speed of visual and auditory reactions. Name the indicator that most objectively reflects the stress of the body's thermoregulation processes:

Температура шкіри Skin Temperature

Кількість дихальних актів за хвилину Number of breaths per minute

Потовиділення за хвилину Sweat per minute

Швидкість зорової та слухової реакцій Speed of visual and auditory reactions

Пульс і артеріальний тиск Pulse and blood pressure

66 / 200
У хворої 58-ми років після 10 років ме-нопаузи почалося рясна маткова кровотеча. В ході бімануального дослідження й огляду за допомогою дзеркал, окрім рясних кров’янистих виділень, іншої патології не виявлено. Яке захворювання можна припустити? A 58-year-old patient started profuse uterine bleeding after 10 years of menopause. During a bimanual examination and examination using mirrors, in addition to profuse bloody discharge , no other pathology was detected. What disease can be assumed?

Міома матки Uterine myoma

Неповний аборт Incomplete abortion

Рак тіла матки Cancer of the uterine body

Геморагічна метропатія Шредера Schroeder's hemorrhagic metropathy

Порушення менструального циклу клімактеричного характеру Disruption of the menstrual cycle of a climacteric character

67 / 200
У дівчинки 12-ти років на шкірі обличчя симптом ’’метелика” на руках рожево-червоні плями з білувато-сірими лусочками, спостерігаються випадіння волосся та біль у суглобах, стійка артеріальна гіпертензія, протеїнурія, еритроцитурія, лейкоцитурія, азотемія. Яке захворювання сполучної тканини можна припустити в першу чергу? A 12-year-old girl has a 'butterfly' symptom on her face, pink-red spots with whitish-gray scales on her hands, hair loss and joint pain , persistent arterial hypertension, proteinuria, erythrocyturia, leukocyturia, azotemia. What connective tissue disease can be assumed first of all?

Дерматоміозит Dermatomyositis

Ювенільний ревматоїдний артрит Juvenile rheumatoid arthritis

Склеродермія Scleroderma

Ревматизм Rheumatism

Системний червоний вовчак Systemic lupus erythematosus

68 / 200
У хлопчика 14-ти років із загостренням вторинного обструктивного пієлонефриту із сечі виділена синьогнійна паличка в титрі 1000000 мікробних тіл на 1 мл. Який антибактеріальний препарат найбільш доцільно призначити в даному випадку? A 14-year-old boy with an exacerbation of secondary obstructive pyelonephritis was isolated from the urine of Pseudomonas aeruginosa with a titer of 1,000,000 microbial bodies per 1 ml. Which antibacterial drug is the most appropriate to prescribe in this case case?

Ампіцилін Ampicillin

Цефазолін Cefazolin

Левоміцетин Levomycetin

Ципрофлоксацин Ciprofloxacin

Азитроміцин Azithromycin

69 / 200
Пацієнтка 22-х років звернулася до жіночої консультації зі скаргами на затримку менструації протягом 1,5 місяців, нудоту, втомлюваність, сонливість, дратівливість. В ході огляду на її обличчі та сосках виявлена виражена пігментація. З боку внутрішніх органів патології не виявлено. Під час огляду в дзеркалах визначається ціаноз слизової оболонки піхви та шийки матки; в ході бімануального дослідження - збільшення матки, її гіперантефлексія, асиметрія. З чим найімовірніше пов’язані перераховані скарги та дані бімануального дослідження? A 22-year-old patient turned to a women's consultation with complaints of delayed menstruation for 1.5 months, nausea, fatigue, drowsiness, irritability. During the examination of her pronounced pigmentation was found on the face and nipples. No pathology was found on the part of the internal organs. During the examination in the mirrors, cyanosis of the mucous membrane of the vagina and cervix was determined; during the bimanual examination - the enlargement of the uterus, its hyperanteflexion, asymmetry. What are the most likely reasons for the listed complaints and bimanual research data?

Захворювання шлунково-кишкового тракту Diseases of the gastrointestinal tract

Порушення менструального циклу Menstrual cycle disorder

Маткова вагітність Uterine pregnancy

Ектопічна вагітність Ectopic pregnancy

Пухлина матки Uterine tumor

70 / 200
У хворої 10-ти років на тлі хронічної ниркової недостатності продовжує прогресувати анемічний синдром. Вкажіть препарат вибору для патогенетичного лікування вказаного синдрому: Anemic syndrome continues to progress in a 10-year-old patient against the background of chronic renal failure. Specify the drug of choice for the pathogenetic treatment of the specified syndrome:

Еритропоетин Erythropoietin

Феррум-лек Ferrum-lek

Ціанокобаламін Cyanocobalamin

Фолієва кислота Folic acid

Еритроцитарна маса Erythrocyte mass

71 / 200
У пацієнта 2-х місяців у пахово-стегнових складках спостерігаються чітко окреслені гострозапальні вогнища у вигляді плям, дещо припіднятих над довколишніми ділянками через набряк шкіри. Висипання з’явилися протягом тижня. Везикуляція та попрілість не спостерігались. Яким буде попередній діагноз? A 2-month-old patient has clearly defined acute inflammatory foci in the form of spots in the inguinal-femoral folds, slightly raised above the surrounding areas due to skin edema. Rashes appeared during the week. Vesiculation and flatulence were not observed. What will be the preliminary diagnosis?

Дитяча екзема Children's eczema

Псоріаз Psoriasis

Мікоз гладенької шкіри Mycosis of smooth skin

Пелюшковий дерматит Diaper dermatitis

Ускладнений перебіг корости Complicated course of scabies

72 / 200
Працівник перебував 16 днів на стаціонарному лікуванні з приводу пневмонії. Який порядок видачі листка непрацездатності в цьому випадку? The employee was hospitalized for 16 days due to pneumonia. What is the procedure for issuing a sick leave in this case?

Лікарем-куратором із дня надходження до стаціонару максимально до 10 днів By the supervising physician from the day of admission to the hospital for a maximum of 10 days

Лікарем-куратором спільно з завідувачем відділення за весь період лікування By the supervising physician together with the head of the department for the entire period of treatment

За висновком лікарсько-консультативної комісії за весь період лікування According to the conclusion of the medical advisory commission for the entire period of treatment

Лікарем-куратором із дня надходження до стаціонару максимально до 5 днів By the supervising physician from the day of admission to the hospital for a maximum of 5 days

Лікарем-куратором із дня надходження до стаціонару максимально до 14 днів By the supervising physician from the day of admission to the hospital for a maximum of 14 days

73 / 200
Хвора скаржиться на біль внизу живота, який посилюється під час місячних і статевих контактів, та іррадіює в піхву. Під час піхвового дослідження - ззаду від матки виявляються щільні, вузлуваті, болючі утворення. Який найбільш імовірний діагноз? The patient complains of pain in the lower abdomen, which intensifies during menstruation and sexual intercourse, and radiates into the vagina. During a vaginal examination, dense, nodular cysts are found behind the uterus , painful formations. What is the most likely diagnosis?

Параметрит Parameter

Периметрит Perimetrit

Аденоміоз Adenomyosis

Ретроцервікальний ендометріоз Retrocervical endometriosis

Хронічне запалення придатків матки Chronic inflammation of the uterine appendages

74 / 200
5-й день. Турбує підвищена до 39-40o 74. C температура тіла, настирливий, частий, глибокий, малопродуктивний кашель, світлобоязнь, закладеність носа. Об’єктивно: одутлість обличчя, слизова оболонка рота пухка, геморагічна енантема на м’якому піднебінні, рештки плям Філатова-Копліка. Який найбільш імовірний діагноз? Day 5. Body temperature raised to 39-40o 74.C, annoying, frequent, deep, unproductive cough, photophobia, nasal congestion is worrying. objectively: puffiness of the face, loose mucous membrane of the mouth, hemorrhagic enanthema on the soft palate, remnants of Filatov-Koplik spots. What is the most likely diagnosis?

Скарлатина Scarlatina

Аденовірусна інфекція Adenovirus infection

Грип Flu

Кір Measles

Краснуха Krasnukha

75 / 200
Під час обстеження дитини виявлено, що вона може тривало спостерігати за яскравою іграшкою, усміхається. В положенні на животі - підіймає та утримує голову. Не сидить. Вік дитини, виходячи з її психосоматичного розвитку, відповідає: During the examination of the child, it was found that she can observe a bright toy for a long time, smiles. In the stomach position - raises and holds her head. Does not sit. Age of the child, based on her psychosomatic development, answers:

7 місяцям 7 months

4 місяцям 4 months

5 місяцям 5 months

6 місяцям 6 months

2 місяцям 2 months

76 / 200
В ході вивчення умов праці робітників, зайнятих на виробництві ртутних термометрів, в повітрі робочої зони виявлено пари ртуті в концентраціях, які перевищують гранично допустимі. Вкажіть основний можливий шлях потрапляння ртуті в організм працівника: In the course of studying the working conditions of workers engaged in the production of mercury thermometers, mercury vapors were detected in the air of the working area in concentrations that exceed the maximum permissible. Specify the main possible route of exposure of mercury into the worker's body:

Шлунково-кишковий тракт Gastrointestinal tract

Ушкоджена шкіра Damaged skin

Органи дихання Respiratory organs

Неушкоджена шкіра Intact Skin

Слизові оболонки Mucous membranes

77 / 200
У новонародженого, що народився в результаті стрімких пологів, спостерігається парез м’язів кисті. Хапальний рефлекс відсутній, не вдається викликати долонно-ротовий рефлекс. Чутливість кисті відсутня. Який найбільш імовірний діагноз? A newborn born as a result of rapid delivery has paresis of the muscles of the hand. The grasping reflex is absent, it is not possible to trigger the hand-mouth reflex. The sensitivity of the hand is absent. What is the most likely diagnosis?

Синдром Горнера-Бернара Horner-Bernard syndrome

Парез діафрагми Paresis of the diaphragm

Тотальне ураження плечового сплетення Total lesion of brachial plexus

Парез Дюшена-Ерба Duchen-Erb paresis

Парез Дежерін-Клюмпке Dezherin-Klumpke paresis

78 / 200
Оцінюючи стан здоров’я водіїв та постових автоінспекторів лікарі виявили наявність в крові обстежуваних карбоксигемоглобіну, зниження у них рефлекторних реакцій, порушення активності ряду ферментів. Виявлені порушення здоров’я людей цих професійних категорій імовірніше всього пов’язані з впливом: Assessing the state of health of drivers and on-duty auto inspectors, doctors found the presence of carboxyhemoglobin in the blood of the subjects, a decrease in their reflex reactions, impaired activity of a number of enzymes. Identified health disorders people of these professional categories are most likely related to the influence:

Сірчистого ангідриду Sulfuric anhydride

Ароматичних вуглеводів Aromatic carbohydrates

Оксидів азоту Nitrogen oxides

Оксиду вуглецю Carbon monoxide

Нервово-емоційного напруження Nervous and emotional tension

79 / 200
У новонародженого, що переніс асфіксію у пологах, на 3-тю добу життя з’явилася кровотеча з пупкової ранки. Лабораторно - гіпокоагуляція, тромбоцитопенія, гіпотромбінемія. Чим зумовлені клініко-лабораторні зміни? On the 3rd day of life, bleeding from the umbilical wound appeared in a newborn who suffered asphyxia during childbirth. Laboratory tests - hypocoagulation, thrombocytopenia, hypothrombinemia. What are the causes clinical and laboratory changes?

Тромбоцитопенічною пурпурою Thrombocytopenic purpura

ДВЗ -синдромом DVT syndrome

Травмою пупкової судини Injury of the umbilical vessel

Вродженою ангіопатією Congenital angiopathy

Геморагічною хворобою новонароджених Hemorrhagic disease of newborns

80 / 200
-ти років скаржиться на підвищення температури тіла до 38o 80. C болі внизу живота і виділення з піхви. Три дні тому було проведено штучний аборт в 10 тижнів вагітності. PV шийка матки чиста, матка дещо збільшена в розмірах, болюча. Придатки матки не визначаються. Склепіння глибокі, безболісні. Виділення з піхви гнійно-кров’яні. Який найбільш імовірний діагноз? -year-old complains of an increase in body temperature to 38o 80. C pain in the lower abdomen and discharge from the vagina. Three days ago, an artificial abortion was performed at 10 weeks of pregnancy. PV of the cervix is clear, the uterus is slightly enlarged, painful. The appendages of the uterus are not defined. The vaults are deep, painless. Vaginal discharge is purulent and bloody. What is the most likely diagnosis?

Пельвіоперитоніт Pelvioperitonitis

Гематометра Hematometer

Післяабортний метроендометрит Postabortion metroendometritis

Перфорація матки після аборту Uterine perforation after abortion

Параметрит Parameter

81 / 200
Хворий 63-х років скаржиться на біль у правій пахвинній ділянці, здуття живота, слабкість, стійкі закрепи більше 7 місяців. Об’єктивно: шкіра бліда, суха. В правій здухвинній ділянці пальпується утворення 5-7 см, малорухоме, неболюче. Аускультативно: підсилення кишкових шумів. У крові: еритроцити - 2,9*1012/л, НЬ-80 г/л, швидкість осідання еритроцитів - 32 мм/год. Кров у калі. Який найбільш імовірний діагноз? A 63-year-old patient complains of pain in the right inguinal region, abdominal distension, weakness, persistent constipation for more than 7 months. Objectively: the skin is pale, dry. A mass of 5-7 cm is palpable in the right iliac region, immobile, painless. Auscultation: increased bowel sounds. In the blood: erythrocytes - 2.9*1012/l, Hb-80 g/l, erythrocyte sedimentation rate - 32 mm/h. Blood in the stool. What is the most likely diagnosis?

Поліп сліпої кишки Cecum polyp

Спастичний коліт Spastic Colitis

Рак сліпої кишки Cancer of the appendix

Хвороба Крона Crohn's disease

Рак правої нирки Right kidney cancer

82 / 200
Хворий 67-ми років скаржиться на задишку, біль у грудях, загальну слабкість. Хворіє 5 місяців. Об’єктивно: температура тіла - 37,3oC, пульс - 96/хв. Над правою легенею голосове тремтіння не визначається, перкуторний звук тупий, дихання не прослуховується. У харкотинні - домішки крові дифузно змішаної зі слизом. Який найбільш імовірний діагноз? A 67-year-old patient complains of shortness of breath, chest pain, general weakness. He has been sick for 5 months. Objectively: body temperature - 37.3oC, pulse - 96/min. Over the right lung, the voice tremor is not determined, the percussion sound is dull, breathing is not heard. In the sputum there are impurities of blood diffusely mixed with mucus. What is the most likely diagnosis?

Бронхоектатична хвороба Bronchoectatic disease

Вогнищевий туберкульоз легенів Focal pulmonary tuberculosis

Рак легенів Lung cancer

Ексудативний плеврит Exudative pleurisy

Великовогнищева пневмонія Large focus pneumonia

83 / 200
Хвора 29-ти років впродовж 2-х місяців скаржиться на болі в лівій половині грудної клітки, кашель, задишку; температура тіла - 39,6oC. Об’єктивно: ліва половина грудної клітки відстає в акті дихання, ослаблення везикулярного дихання та вкорочення перкуторного звуку зліва. Ro-логічно визначається округла тінь у нижній частці лівої легені. Який попередній діагноз? A 29-year-old patient has been complaining of pain in the left half of the chest for 2 months, cough, shortness of breath; body temperature is 39.6oC. Objectively : the left half of the chest lags behind in the act of breathing, weakening of vesicular breathing and shortening of the percussion sound on the left. Ro-logically, a rounded shadow is determined in the lower lobe of the left lung. What is the preliminary diagnosis?

Емпієма плеври Empyema of the pleura

Гнійний плеврит Suppurative pleurisy

Абсцес легені Lung abscess

Рак легені Lung cancer

Хронічна пневмонія Chronic pneumonia

84 / 200
Пацієнтка 45-ти років скаржиться на дискомфорт під час читання, почервоніння краю повік, білі пінисті виділення в кутах очних щілин протягом останніх двох місяців. Об’єктивно спостерігаються гіперемія та потовщення вільного краю повік, розширення вивідних протоків залоз хряща повік. Яким буде діагноз? A 45-year-old female patient complains of discomfort while reading, redness of the edge of the eyelids, white foamy discharge in the corners of the eye sockets for the past two months. Hyperemia is objectively observed and thickening of the free edge of the eyelids, expansion of the excretory ducts of the eyelid cartilage glands. What will be the diagnosis?

Аденовірусний кон’юнктивіт Adenoviral conjunctivitis

Гострий дакріоаденіт Acute dacryoadenitis

Хронічний каналікуліт Chronic canaliculitis

Бленорейний кон’юнктивіт Blenorrhoeic conjunctivitis

Мейбомієвий блефарит Meibomian blepharitis

85 / 200
Дівчинці 8 місяців; народилася недоношеною. Під час огляду: спостерігаються задуха, тахікардія, гепатоспленомегалія, відставання в фізичному розвитку, ціаноз кінцівок. Визначається парастернальний серцевий горб, у II міжребер’ї зліва вислуховується систолодіастолічний шум, артеріальний тиск - 90/0 мм рт.ст. Яке захворювання можна припустити? The girl is 8 months old; she was born prematurely. During the examination: suffocation, tachycardia, hepatosplenomegaly, delayed physical development, cyanosis of the limbs are observed. A parasternal cardiac hump is detected, in II a systolic diastolic murmur is heard in the left intercostal space, blood pressure is 90/0 mm Hg. What disease can be assumed?

Коарктація аорти Coarctation of the aorta

Стеноз аортального клапана Aortic valve stenosis

Відкрита артеріальна протока Open ductus arteriosus

Незарощення міжшлуночкової перегородки Non-occlusion of the interventricular septum

Стеноз легеневої артерії Pulmonary artery stenosis

86 / 200
У потерпілого опікова травма 15% поверхні тіла II-III ст. На 20-ту добу після травми у хворого спостерігаються різке підвищення температури тіла, загальна слабкість, часте везикулярне дихання, загострення рис обличчя, артеріальний тиск - 90/50 мм рт.ст., пульс - 112/хв. Яке ускладнення можна припустити? The victim has a burn injury of 15% of the body surface of the II-III degree. On the 20th day after the injury, the patient has a sharp increase in body temperature, general weakness, frequent vesicular breathing, aggravation of facial features, blood pressure - 90/50 mm Hg, pulse - 112/min. What complications can be assumed?

Гостра інтоксикація Acute intoxication

Пневмонія Pneumonia

Гнійний бронхіт Purulent bronchitis

Сепсис Sepsis

Анаеробна інфекція Anaerobic infection

87 / 200
У поточному році лікарями загальної практики міської поліклініки було направлено до стаціонару 11 хворих з ішемічною хворобою серця. У 3-х випадках діагноз не підтвердився. Яке управлінське рішення найдоцільніше прийняти в цьому випадку? In the current year, general practitioners of the city polyclinic sent 11 patients with coronary heart disease to the hospital. In 3 cases, the diagnosis was not confirmed. What management decision is the most appropriate to take in this case?

Аналіз якості дiагностичних досліджень Analysis of the quality of diagnostic studies

Аналіз матеріально-технічної бази поліклініки Analysis of the material and technical base of the polyclinic

Аналіз кожного випадку розходження діагнозу Analysis of each case of diagnosis discrepancy

Аналіз якості диспансерного спостереження Analysis of the quality of dispensary observation

Аналіз рівня кваліфікації лікарів поліклініки Analysis of the qualification level of polyclinic doctors

88 / 200
В ході лабораторного дослідження зразку свинини в 24-х зрізах виявлена 1 мертва трихінела. Таке м’ясо слід передати для: During the laboratory examination of a pork sample, 1 dead trichinella was found in 24 slices. Such meat should be transferred to:

C в товщі м’яса з подальшою експозицією C in the thickness of the flesh with further exposure

Технічної утилізації Technical disposal

Переробки для виготовлення варених ковбас Заморожування до температури 10o Processes for making cooked sausages Freezing to a temperature of 10o

Реалізації через мережу закладів громадського харчування Realizations through a network of catering establishments

Реалізації без обмежень Unlimited implementations

89 / 200
діб 89. У чоловіка 43-х років під час проведення медичного огляду об’єктивно виявлені блідість шкіри та слизових оболонок, згла-дженість сосочків язика, поперечні борозни на нігтях, тріщини в кутиках рота, тахікардія. Вміст гемоглобіну в крові - 90 г/л, анізоцитоз, пойкілоцитоз. Найімовірнішим спричинювальним фактором даного стану є недостатнє надходження до організму: day 89. In a 43-year-old man, during a medical examination, pallor of the skin and mucous membranes, smoothness of the papillae of the tongue, transverse furrows on nails, cracks in the corners of the mouth, tachycardia. The hemoglobin content in the blood is 90 g/l, anisocytosis, poikilocytosis. The most likely causative factor of this condition is insufficient supply to the body:

Заліза Iron

Міді Copper

Магнію Magnesium

Селену Selena

Цинку Zinc

90 / 200
У хворої 52-х років спостерігається ураження слизової оболонки в кутиках рота з утворенням тріщин, ерозій та виразок, вертикальні тріщини на губах при їх зімкненні (хейлоз), зміни язика (глосит), ангулярний стоматит, себорейне лущення шкіри навколо рота та на крилах носа, перикорнеальна ін’єкція. Вищеперераховані симптоми є характерними для: A 52-year-old patient has lesions of the mucous membrane in the corners of the mouth with the formation of cracks, erosions and ulcers, vertical cracks on the lips when they are closed (cheilosis), changes tongue (glossitis), angular stomatitis, seborrheic peeling of the skin around the mouth and on the wings of the nose, pericorneal injection. The above symptoms are characteristic of:

РР-гіповітамінозу PP-hypovitaminosis

Б2-гіповітамінозу B2 hypovitaminosis*

Б1-гіповітамінозу B1 hypovitaminosis

C-гіповітамінозу C-hypovitaminosis

А-гіповітамінозу A hypovitaminosis

91 / 200
Впродовж тривалого часу проводиться вивчення тенденцій у змінах показників загальної смертності різних груп населення, які проживають на різних адміністративних територіях. Який зі статистичних методів може бути використано з цією метою? Over a long period of time, trends in changes in total mortality rates of various population groups living in different administrative territories have been studied. Which of the statistical methods can be used for this purpose?'

Аналіз динамічних рядів Analysis of dynamic series

Аналіз стандартизованих показників Analysis of standardized indicators

Аналіз рівнів відносних величин Analysis of levels of relative values

Оцінка вірогідності різниці показників Estimation of the probability of the difference of indicators

Кореляційно-регресивний аналіз Correlation-regression analysis

92 / 200
У хворого 27-ми років внаслідок падіння з висоти стався перелом правої стегнової кістки в нижній третині з випинанням під шкіру дистального фрагменту. Кінцівка тепла, пацієнт пальцями стопи рухає. Лікар швидкої допомоги відмовився від шини Дітеріхса і наклав іммобілізацію трьома драбинчастими шинами на зігнуту в коліні кінцівку. Яке можливе ускладнення він цим попередив? A 27-year-old patient suffered a fracture of the right femur in the lower third as a result of a fall from a height with protrusion of the distal fragment under the skin. The limb is warm, the patient moves his toes. The emergency doctor abandoned the Dieterichs splint and applied immobilization with three ladder splints to the limb bent at the knee. What possible complication did he prevent by this?

Вторинне зміщення кісткових фрагментів Secondary displacement of bone fragments

Інтерпозицію м’язів та перфорацію шкіри Muscle interposition and skin perforation

Розвиток травматичного шоку Development of traumatic shock

Пошкодження підколінного судинно-нервового пучка Damage of the popliteal neurovascular bundle

Розвиток жирової емболії Development of fat embolism

93 / 200
Хворий 28-ми років надійшов до відділення інтенсивної терапії зі скаргами на біль у животі, блювання, двоїння в очах. Об’єктивно: дезорієнтований, зіниці розширені, реакція на світло ослаблена, шкіра суха, гіперемована, незначний ціаноз, артеріальний тиск - 90/60 мм рт.ст., пульс -100/хв. З анамнезу відомо, що за два дні до госпіталізації вживав алкоголь невідомого походження. Специфічна терапія полягає у внутрішньовенному введенні: A 28-year-old patient came to the intensive care unit with complaints of abdominal pain, vomiting, double vision. Objectively: disoriented, dilated pupils, reaction weak to light, the skin is dry, hyperemic, slight cyanosis, blood pressure - 90/60 mm Hg, pulse -100/min. It is known from the anamnesis that two days before hospitalization he consumed alcohol of unknown origin. The specific therapy consists of intravenous inputs:

Тіаміну Thiamine

Етанолу Ethanol

Омепразолу Omeprazole

40% розчину глюкози 40% glucose solution

Гідрокарбонату натрію Sodium bicarbonate

94 / 200
Хвора 29-ти років скаржиться на відсутність менструації протягом року, швидку втомлюваність, гіпотонію з синкопальними станами зниження пам’яті, сухість шкіри. З анамнезу відомо, що 1,5 роки тому в ІІ періоді пологів була значна кровотеча, проведено екстирпацію матки, гемотрансфузію, перебувала на ШВЛ 3 доби. Який найбільш імовірний діагноз? A 29-year-old patient complains of the absence of menstruation for a year, rapid fatigue, hypotension with syncopal states, memory loss, dry skin. From the anamnesis, it is known that 1 .5 years ago in the second period of labor, there was significant bleeding, extirpation of the uterus, hemotransfusion was performed, she was on mechanical ventilation for 3 days. What is the most likely diagnosis?

Адреногенітальний синдром Adrenogenital syndrome

Синдром Аронце дель Кастілліо Aronze del Castillo Syndrome

Синдром Кіарі-Фроммеля Chiari-Frommel syndrome

Синдром Фарбса-Олбрайта Farbs-Albright syndrome

Синдром Шиєна Schien syndrome

95 / 200
У хворої 50-ти років впродовж півтора місяця спостерігається стабільне наростання жовтяниці, анемії з періодичними підйомами температури тіла. Під час пальпації виявлено збільшений і безболісний жовчний міхур. Розвиток якого захворювання можна припустити? In a 50-year-old patient, a stable increase in jaundice and anemia with periodic rises in body temperature has been observed for one and a half months. During palpation, an enlarged and painless gallbladder was found. The development of which can the disease be assumed?

Первинний склерозивний холангіт Primary sclerosing cholangitis

Рак головки підшлункової залози Cancer of the head of the pancreas

Рак жовчного міхура Gall bladder cancer

Жовчнокам’яна хвороба Cholelithiasis

Ехінококоз печінки Echinococcosis of the liver

96 / 200
До гінеколога звернулася жінка 54-х років зі скаргами на кров’яні виділення з піхви протягом 1 місяця. Остання менструація була 5 років тому. В ході гінекологічного огляду патології не виявлено. Якими будуть дії лікаря? A 54-year-old woman came to the gynecologist with complaints of vaginal bleeding for 1 month. The last menstruation was 5 years ago. During a gynecological pathology examination not detected. What will be the actions of the doctor?

Взяти мазок для цитологічного дослідження Take smear for cytological examination

УЗД Ultrasound

Призначити симптоматичну терапію Prescribe symptomatic therapy

Кольпоскопія Colposcopy

Фракційне діагностичне вишкрібання стінок порожнини матки Fractional diagnostic scraping of the walls of the uterine cavity

97 / 200
Лікаря швидкої допомоги викликано до постраждалого. Циркулярною пилкою зроблено відрив лівої кисті на рівні променевозап’ястного суглоба у робітника 25-ти років. Має місце артеріальна кровотеча. Де треба накласти джгут для тимчасової зупинки артеріальної кровотечі? An emergency doctor was called to the victim. A 25-year-old worker was amputated with a circular saw at the level of the radiocarpal joint of the left hand. There is arterial bleeding. Where necessary apply a tourniquet to temporarily stop arterial bleeding?

- -

Нижня третина передпліччя Lower third of forearm

Середня третина плеча Middle third shoulder

Верхня третина передпліччя Upper third of forearm

Ділянка ліктьового суглоба Elbow joint area

98 / 200
У хворого 35-ти років під час трансфузії 400 мл еритроцитарної маси розвинулися загальний неспокій, короткочасне збудження, біль за грудиною, у попереку, задишка, ціаноз, тахікардія - 110/хв., артеріальний тиск знизився до 90/40 мм рт.ст. Який найбільш імовірний діагноз? A 35-year-old patient developed general restlessness, short-term excitement, pain behind the sternum, in the lower back, shortness of breath, cyanosis, tachycardia during transfusion of 400 ml of erythrocyte mass - 110/min., blood pressure decreased to 90/40 mm Hg. What is the most likely diagnosis?

Тромбоемболія легеневої артерії Thromboembolism of the pulmonary artery

Синдром масивних гемотрансфузій Massive blood transfusion syndrome

Алергічна реакція Allergic reaction

Цитратна інтоксикація Citrate intoxication

Гемотрансфузійний шок Hemotransfusion shock

99 / 200
Хворий 38-ми років скаржиться на гнійне виділення із лівої половини носа, утруднене носове дихання, головний біль, важкість в області лівої щоки, підвищення температури тіла до 37,5oC. Хворіє впродовж 6-ти днів. Захворювання зв’язує з переохолодженням. Об’єктивно: болісність при пальпації в області лівої щоки. Слизова оболонка лівої половини носової порожнини гіперемована, набухла, в середньому носовому ході - гнійна полоска. Який найбільш імовірний діагноз? A 38-year-old patient complains of purulent discharge from the left half of the nose, difficulty breathing through the nose, headache, heaviness in the left cheek area, an increase in body temperature to 37, 5oC. Sick for 6 days. The disease is associated with hypothermia. Objectively: pain upon palpation in the area of the left cheek. The mucous membrane of the left half of the nasal cavity is hyperemic, swollen, and there is a purulent strip in the middle nasal passage. What is the most likely diagnosis ?

Лівобічний гострий етмоїдит Left-sided acute ethmoiditis

Гострий риніт (нежить) Acute rhinitis (runny nose)

Лівобічний гострий фронтит Left-sided acute frontitis

Лівобічний гострий гайморит Left-sided acute sinusitis

Викривлення носової перегородки вліво Curvation of the nasal septum to the left

100 / 200
У хлопчика 11-ти років під час профілактичного медичного огляду визначали вид постави. Встановлено, що у дитини плечі похилені і зведені вперед, голова нахилена вперед, грудна клітина сплощена, живіт опуклий. В хребті спостерігається збільшення глибини шийного та поперекового вигинів. Який вид постави виявлений у дитини? The posture of an 11-year-old boy was determined during a preventive medical examination. It was found that the child's shoulders are bent and bent forward, the head is tilted forward, the chest is flattened , the stomach is bulging. There is an increase in the depth of the cervical and lumbar curves in the spine. What type of posture is found in the child?

Нормальна Normal

Виправлена Fixed

Лордоз Lordosis

Сутулувата Shunted

Кіфоз Kyphosis

101 / 200
Хвора 18-ти років через 2 тижні після ангіни скаржиться на підвищення температури тіла, біль та припухлість у колінних суглобах, висип у вигляді червоних кілець на гомілках. Через кілька днів почав турбувати біль у гомілковостопних суглобах, потім - у ліктях. Для якого захворювання характерні такі симптоми? 2 weeks after angina, an 18-year-old patient complains of an increase in body temperature, pain and swelling in the knee joints, a rash in the form of red rings on the lower legs. After a few days, he began to suffer from pain in the ankle joints, then in the elbows. What disease is characterized by such symptoms?

Токсико-алергічний дерматит Toxic-allergic dermatitis

Гостра ревматична лихоманка Acute rheumatic fever

Деформівний остеоартроз Deformative osteoarthritis

Ревматоїдний артрит Rheumatoid arthritis

Реактивний артрит Reactive arthritis

102 / 200
Хвора 21-го року звернулася до жіночої консультації зі скаргами на значне нездужання, сильний біль у ділянці вульви, подразнення, свербіння, прискорене сечовипускання. З анамнезу з’ясовано, що жінка 10 днів тому мала позашлюбний статевий контакт. Під час огляду зовнішніх статевих органів виявлено: на гіперемованій, набряклій слизовій оболонці великих статевих губ розташовані численні пухирці 2-3 мм в діаметрі. Після проведення додаткових методів діагностики виявлено вірус простого герпесу. Який препарат використовується для лікування даного захворювання? A 21-year-old patient turned to a women's consultation with complaints of significant malaise, severe pain in the vulva, irritation, itching, accelerated urination. From the anamnesis, it was found , that the woman had extramarital sexual contact 10 days ago. During the examination of the external genitalia, it was found: numerous blisters 2-3 mm in diameter are located on the hyperemic, swollen mucous membrane of the labia majora. After conducting additional diagnostic methods, the herpes simplex virus was detected. What drug used to treat this disease?

Сульфадимізин Sulfadimyzin

Ацикловір Acyclovir

Метронідазол Metronidazole

Прогестерон Progesterone

Тетрациклін Tetracycline

103 / 200
Хвора 47-ми років вагою 100 кг звернулася зі скаргами на ниючий біль в колінних та кульшових суглобах, який виникає під час рухів і в стані спокою, підсилюється під вечір та при фізичному навантаженні. Об’єктивно: колінні суглоби деформовані, під час пальпації - біль у внутрішній поверхні в місці проекції суглобової щілини, рухи супроводжуються хрускотом. Протягом 16 років хворіє на виразкову хворобу ДПК Який засіб симптоматичної терапії є найбільш доцільним у даній ситуації? A 47-year-old patient weighing 100 kg complained of aching pain in the knee and hip joints, which occurs during movement and at rest, intensifies in the evening and during physical exertion. Objectively: the knee joints are deformed, during palpation - pain in the inner surface at the point of projection of the joint gap, movements are accompanied by crunching. For 16 years, he has been suffering from peptic ulcer disease of the DPK. What means of symptomatic therapy is the most appropriate in this situation?

Ібупрофен Ibuprofen

Піроксикам Piroxicam

Целекоксиб Celecoxib

Індометацин Indomethacin

Диклофенак натрію Diclofenac sodium

104 / 200
До жіночої консультації звернулася жінка 24-х років зі скаргами на затримку менструації на 10 тижнів. В анамнезі 2 спонтанних викидні. В ході ультразвукового дослідження виявлено одне плідне яйце, яке відповідає 6-7 тижням вагітності, серцебиття плода не визначається. Яка подальша тактика в даній ситуації? A 24-year-old woman came to the women's consultation with complaints of a delay in menstruation for 10 weeks. She has 2 spontaneous miscarriages in her history. During the ultrasound examination, one fertile egg was found, which corresponds to 6-7 weeks of pregnancy, the heartbeat of the fetus is not determined. What are the further tactics in this situation?

Застосування утеротонічних засобів Use of uterotonic means

Вишкрібання стінок порожнини матки Scraping the walls of the uterine cavity

Призначення антибактеріальної терапії Prescription of antibacterial therapy

Призначення гормональної терапії Prescription of hormone therapy

Спостереження протягом 1 тижня з наступним УЗД-контролем Observation for 1 week followed by ultrasound control

105 / 200
Хворого 47-ми років почав турбувати стискальний біль за грудиною, що виникає під час ходи на 700-800 м. Один раз на тиждень випиває 2 л пива. Артеріальна гіпертензія впродовж останніх 7-ми років. Об’єктивно: пульс - 74/хв., артеріальний тиск -120/80 мм рт.ст. При проведенні ВЕМ з навантаженням 75 Вт зареєстровано депресію сегмента ST на 2 мм нижче ізолінії у V4 — V6. Який найбільш імовірний діагноз? A 47-year-old patient began to be bothered by squeezing pain behind the sternum, which occurs during a walk of 700-800 m. He drinks 2 liters of beer once a week. Arterial hypertension for the past 7 years. Objectively: pulse - 74/min., blood pressure -120/80 mm Hg. When carrying out VEM with a load of 75 W, depression of the ST segment 2 mm below the isoline in V4 — V6 was registered . What is the most likely diagnosis?

Вегето-судинна дистонія за гіпертонічним типом Vegeto-vascular dystonia of the hypertensive type

Стенокардія напруги, III функціональний клас Tension angina, III functional class

Стенокардія напруги, II функціональний клас Tension angina, II functional class

Стенокардія напруги, IV функціональний клас Tension angina, IV functional class

Алкогольна кардіоміопатія Alcoholic cardiomyopathy

106 / 200
Хворий 43-х років через 2 місяці після перенесеного тонзиліту госпіталізований у терапевтичне відділення зі скаргами на задишку, біль в ділянці серця, запаморочення, серцебиття. Об’єктивно: загальний стан важкий. Частота дихання - 35/хв., пульс - 100/хв., аритмічний, (екстрасистолія). Артеріальний тиск - 145/60 мм рт.ст. Тони серця аритмічні (екстрасистолія) послаблення I тону, систолічний шум над верхівкою. Печінка збільшена. ЕКГ: синусова тахікардія, поодинокі шлуночкові екстрасистоли. ЕхоКГ порожнини серця в межах норми, ФВ- 50%. Який найбільш імовірний діагноз? A 43-year-old patient was hospitalized 2 months after tonsillitis with complaints of shortness of breath, heart pain, dizziness, palpitations. Objectively: general condition is serious. Respiratory rate - 35/min., pulse - 100/min., arrhythmic, (extrasystole). Blood pressure - 145/60 mm Hg. Heart sounds are arrhythmic (extrasystole), weakening of the first sound, systolic murmur over the apex The liver is enlarged. ECG: sinus tachycardia, single ventricular extrasystoles. Echocardiography of the heart cavity is within normal limits, FV-50%. What is the most likely diagnosis?

Інфекційний перикардит Infectious pericarditis

Дилятаційна кардіоміопатія Dilated cardiomyopathy

Міокардитичний кардіофіброз Myocardial cardiofibrosis

Синдром Дресслера Dressler Syndrome

Інфекційний міокардит Infectious myocarditis

107 / 200
33-х років гостра крововтрата (еритроцити - 2,2*1012 107. /л, Hb- 55 г/л), група крові A(II) Rh(+). Йому помилково проведено трансфузію донорської еритромаси AB(IV) Rh(+). Через годину з’явилися відчуття тривоги, біль у попереку, животі. Пульс - 134/хв., артеріальний тиск - 100/65 мм рт.ст., температура тіла - 38,6oC. При катетеризації сечового міхура отримано 12 мл/год сечі темно-коричневого кольору. Яке ускладнення виникло у хворого? 33-year-old acute blood loss (erythrocytes - 2.2*1012 107./l, Hb- 55 g/l), blood group A(II) Rh(+). He was mistakenly transfused with donor erythromass AB(IV) Rh(+). An hour later, feelings of anxiety, pain in the lower back, abdomen appeared. Pulse - 134/min., blood pressure - 100/65 mm Hg. st., body temperature - 38.6oC. During bladder catheterization, 12 ml/h of dark brown urine was obtained. What complication did the patient experience?

Інфекційно-токсичний шок Infectious-toxic shock

Кардіальний шок Cardiac shock

Цитратна інтоксикація Citrate intoxication

Гостра ниркова недостатність Acute renal failure

Алергічна реакція на донорську еритро-масу Allergic reaction to donor erythrocyte mass

108 / 200
У чоловіка 45-ти років протягом останніх двох тижнів розвинулася підвищена активність, став балакучим, ейфорі-чним, мало спав, стверджував, що може ’’врятувати людство” здатен вирішити проблему раку та СНЩу, роздавав гроші незнайомцям. Оберіть найбільш імовірний діагноз: In the last two weeks, a 45-year-old man developed increased activity, became talkative, euphoric, slept little, claimed that he could 'save humanity' able to solve the problem of cancer and TMJ, gave money to strangers. Choose the most likely diagnosis:

Панічний розлад Panic Disorder

Маніакальний напад Manic attack

Ажитована депресія Alive Depression

Кататонічне збудження Catatonic excitement

Шизоафективний розлад Schizoaffective disorder

109 / 200
Хворий 44-х років під час сільськогосподарських робіт отримав рану в ділянці правої гомілки. Профілактичного щеплення проти правця не отримував протягом останніх 20-ти років. Яку профілактику правця слід провести? A 44-year-old patient received a wound in the right lower leg during agricultural work. He did not receive a preventive vaccination against tetanus for the past 20 years. What tetanus prevention should be done conduct?

Специфічну - активно-пасивна імунізація Specific - active-passive immunization

Екстрену, неспецифічну (первинна хірургічна обробка рани), специфічну (активнопасивна імунізація) Emergency, non-specific (primary surgical treatment of the wound), specific (active-passive immunization)

Потерпілому ввести 3000 АО протиправцевої сироватки Inject 3000 AU of anti-tetanus serum to the victim

Виконати первинну хірургічну обробку рани Perform initial surgical treatment of the wound

Планову, шляхом введення правцевого анатоксину Planned, by introducing tetanus toxoid

110 / 200
У післяопераційного хворого, якому 6 днів тому проведено видалення нижньої частки лівої легені в зв’язку з бактеріальною деструкцією і розвитком пневмотораксу, наросла задишка до 35/хв., раО2 знизився до 70 мм рт.ст., появився кашель, виділення великої кількості харкотиння. Під час аускультації правої та лівої легень - маса різнокаліберних хрипів, різко ослаблене дихання. Яка ступінь дихальної недостатності у цього хворого? In a postoperative patient who had the lower lobe of the left lung removed 6 days ago due to bacterial destruction and the development of pneumothorax, shortness of breath increased to 35/min., raO2 decreased to 70 mm Hg, a cough appeared, the release of a large amount of sputum. During auscultation of the right and left lungs - a mass of various caliber rales, sharply weakened breathing. What is the degree of respiratory failure in this patient?

III ступінь III degree

II ступінь II degree

IV ступінь IV degree

I ступінь I degree

0 ступінь 0 degree

111 / 200
Чоловік 27-ми років скаржиться на задишку, свербіж і відчуття набряку обличчя, що виникли 25 хвилин тому. Їв раків та пив пиво. Стан прогресивно погіршується. Об’єктивно: збуджений, обличчя одутле, щоки та губи збільшені у розмірі. Частота дихання - 28/хв., чутно свистячі хрипи на вдиху та видиху, при аускультації легень провідні шуми, максимум над грудниною. Тони серця гучні, частота серцевих скорочень (пульсу) - 108/хв., артерiальний тиск -150/90 мм рт.ст. Найбільш імовірною причиною задишки є: A 27-year-old man complains of shortness of breath, itching and a feeling of facial swelling that occurred 25 minutes ago. He ate crayfish and drank beer. The condition is getting progressively worse. About' objectively: excited, the face is swollen, the cheeks and lips are enlarged in size. The respiratory rate is 28/min., whistling rales are audible during inhalation and exhalation, when auscultating the lungs, conducting noises, maximum above the sternum. Heart sounds are loud, heart rate (pulse) - 108/min., arterial pressure -150/90 mm Hg. The most likely cause of shortness of breath is:

Спазм бронхів Bronchi spasm

Нейроциркуляторна дистонія Neurocirculatory dystonia

Гіпертонічний криз Hypertensive crisis

Набряк гортані Swelling of the larynx

Алкогольне сп’яніння Alcohol intoxication

112 / 200
У новонародженого від I вагітності з масою 3500 г з першої доби спостерігаються жовтяниця, млявість, зниження рефлексів. Об’єктивно: жовтяничність шкіри II ступеня із шафрановим відтінком, печінка +2 см, селезінка +1 см. Сеча та випорожнення жовті. У крові: НЬ-100 г/л, еритроцити - 3,2 • 1012/л, лейкоцити - 18, 7 • 109 /л, кров матері 0(I) Rh(+), кров дитини А(Іі) Rh(-), білірубін -170 мкмоль/л, фракція непряма. Рівень АЛТ, АСТ - у нормі. Яке захворювання імовірно у дитини? A newborn from the first pregnancy with a weight of 3500 g has jaundice, lethargy, decreased reflexes from the first day. Objectively: skin jaundice of the second degree with a saffron tint, liver +2 cm, spleen +1 cm. Urine and stools are yellow. Blood: Hb-100 g/l, erythrocytes - 3.2 • 1012/l, leukocytes - 18.7 • 109 /l, mother's blood 0(I) Rh(+), blood of the child A(II) Rh(-), bilirubin -170 μmol/l, indirect fraction. The level of ALT, AST is normal. What disease is likely in the child?

Гемолітична хвороба новонародженого, АВ0-конфлікт Hemolytic disease of the newborn, AB0-conflict

Фізіологічна жовтяниця Physiological jaundice

Гемолітична хвороба новонародженого, Rh-конфлікт Hemolytic disease of the newborn, Rh conflict

Атрезія жовчовивідних шляхів Biliary atresia

Внутрішньоутробний гепатит Intrauterine hepatitis

113 / 200
-ти років надійшла зі скаргами на підвищення температури тіла до 37,2o 113. C, часті та болісні сечовипускання, які з’явилися після переохолодження. У сечі: сеча каламутна, питома вага -1012, білок -0,033o /oo, мікроскопія: лейкоцити - 40-45 в п/з, еритроцити - 8-9 в п/з (свіжі), епітелій плаский: 5-8 в п/з, слиз. Який етіологічний фактор у даному випадку найбільш імовірний? year-old woman came with complaints of an increase in body temperature to 37.2o 113. C, frequent and painful urination, which appeared after hypothermia. In the urine: cloudy urine, specific gravity -1012, protein -0.033o /oo, microscopy: leukocytes - 40-45 in p/z, erythrocytes - 8-9 in p/z (fresh), squamous epithelium: 5-8 in p/z , mucus. What is the most probable etiological factor in this case?

Klebsiella pneumoniae Klebsiella pneumoniae

Proteus mirabilis Proteus mirabilis

Staphylococcus aureus Staphylococcus aureus

Candida albicans Candida albicans

Escherichia coli Escherichia coli

114 / 200
Жінка 35-ти років скаржиться на слабкість, часті випорожнення з домішками крові, зменшення ваги тіла на 8 кг за 6 місяців, періодично - підвищення температури до 37,8oC. Об’єктивно: температура тіла -37,5°C, пульс - 86/хв., артеріальний тиск -110/70 мм рт.ст. Шкіра бліда, пальпаторно живіт помірно болючий. У крові: НЬ-92 г/л, швидкість осідання еритроцитів - 35 мм/год. Колоноскопія: слизова прямої та сигмоподібної кишки зерниста, гіперемована, набрякла, нерівномірно потовщена (псевдополіпи), кровить, на її поверхні - ерозії та виразки. Яка патологія найбільш імовірно викликала ураження кишки у хворої? A 35-year-old woman complains of weakness, frequent stools with blood impurities, a decrease in body weight by 8 kg in 6 months, periodically - an increase in temperature up to 37.8oC . Objectively: body temperature -37.5°C, pulse - 86/min., blood pressure -110/70 mm Hg. Skin is pale, palpable abdomen is moderately painful. In blood: Hb-92 g/l, the sedimentation rate of erythrocytes is 35 mm/h. Colonoscopy: the mucous membrane of the rectum and sigmoid colon is granular, hyperemic, swollen, unevenly thickened (pseudopolyps), bleeding, on its surface there are erosions and ulcers. What pathology most likely caused the patient's bowel damage?

Хвороба Крона Crohn's disease

Виразковий коліт Ulcerative colitis

Рак прямої кишки Rectal cancer

Синдром подразненого кишківника Irritable bowel syndrome

Поліпоз кишки Intestinal polyposis

115 / 200
Чоловік 47-ми років є робітником ткацького цеху, має стаж роботи в даному виробництві 15 років, і працює в умовах високочастотного інтенсивного шуму. Під час періодичного медичного огляду йому було поставлено діагноз ’’професійна приглухуватість’.’ Що є підставою для такого діагнозу? A 47-year-old man is a worker in a weaving workshop, has 15 years of experience in this industry, and works in conditions of high-frequency intense noise. During a periodic medical examination, he a diagnosis of 'occupational deafness' was made. What is the basis for such a diagnosis?

Стаж роботи на даному виробництві Work experience at this production

Результати дослідження стану внутрішнього вуха Results of the study of the condition of the inner ear

Результати дослідження показників ЦНС Results of research of CNS indicators

Характеристика шуму на даному виробництві Characteristics of noise at this production

Дані аудіометрії та гігієнічної оцінки умов праці Data of audiometry and hygienic assessment of working conditions

116 / 200
В районі N 30% випадків раку молочної залози і 26% випадків раку шийки матки виявляються в IV стадії. Яке управлінське рішення в цій ситуації слід прийняти? In area N, 30% of breast cancer cases and 26% of cervical cancer cases are found in stage IV. What management decision should be taken in this situation?

Організувати і провести попередні медичні огляди жінок Organize and conduct preliminary medical examinations of women

Організувати і провести періодичні медичні огляди жінок Organize and conduct periodic medical examinations of women

Організувати і провести комплексні медичні огляди жінок Organize and conduct comprehensive medical examinations of women

Організувати і провести тотальні медичні огляди жінок Organize and conduct total medical examinations of women

Організувати і провести цільові медичні огляди жінок Organize and conduct targeted medical examinations of women

117 / 200
Управлінню охорони здоров’я необхідно оцінити ефективність впровадження програм з охорони материнства і дитинства в області. Рівень якого інтегрального показника найдоцільніше використати для такого аналізу? The Department of Health Care needs to assess the effectiveness of the implementation of maternity and childhood care programs in the region. Which integral indicator level is the most appropriate to use for such an analysis?

Смертності немовлят Infant Mortality

Інвалідності дітей Children's disabilities

Захворюваності дітей Children's diseases

Перинатальної смертності Perinatal mortality

Фізичного розвитку Physical development

118 / 200
До лікаря жіночої консультації звернулася жінка, 6-7 тижнів вагітності. З анамнезу з’ясовано, що з 15-ти років хворіє на цукровий діабет у тяжкій формі. За заключенням окуліста ретинопатія II ст. Мала 2 вагітності, які закінчилися мертвонародженням. Вміст цукру в крові -15 ммоль/л. При бімануальному дослідженні встановлено, що матка збільшена (як при вагітності, терміном до 7 тижнів). Яка тактика лікаря? A woman, 6-7 weeks pregnant, turned to a doctor for a women's consultation. From the anamnesis, it was found that she has been suffering from diabetes in a severe form since she was 15 years old. According to the oculist's conclusion, retinopathy of the II stage. She had 2 pregnancies that ended in stillbirth. The blood sugar content is -15 mmol/l. During a bimanual examination, it was found that the uterus is enlarged (as in pregnancy, up to 7 weeks). What are the doctor's tactics?

Консультація ендокринолога Endocrinologist consultation

Пролонгування вагітності під контролем рівня цукру в крові Prolongation of pregnancy under blood sugar control

Госпіталізація в критичні терміни вагітності Hospitalization in critical periods of pregnancy

Консультація генетика Genetic consultation

Показано переривання вагітності Pregnancy termination is shown

119 / 200
Хвора 50-ти років, яка страждає на хворобу Аддісона (виникла після перенесеного туберкульозу легень у юному віці), під час пожежі в квартирі сильно налякалася і знепритомніла. Хвора бліда, холодна, пульс ниткоподібний, частота серцевих скорочень - 120/хв., артеріальний тиск - 60/30 мм рт.ст. Яке ускладнення виникло у хворої? A 50-year-old patient suffering from Addison's disease (occurred after suffering from pulmonary tuberculosis at a young age), was very frightened and fainted during a fire in her apartment. The patient pale, cold, thread-like pulse, heart rate - 120/min., blood pressure - 60/30 mm Hg. What complication did the patient have?

Тампонада серця Cardiac tamponade

Синдром Морганьї-Адамса-Стокса Morganhi-Adams-Stokes syndrome

Гострий інфаркт міокарда Acute myocardial infarction

Гостра наднирникова недостатність Acute adrenal insufficiency

Тиреотоксичний криз Thyrotoxic crisis

120 / 200
У хворої 50-ти років, яка страждає на гіпертонічну хворобу понад 10 років, на тлі стресу раптово підвищився артеріальний тиск до 200/110 мм рт.ст. Стан супроводжувався тремтінням тіла, головним болем, тахікардією, загальним збудженням, відчуттям жару та сухості в роті. Призначення яких препаратів є найбільш обґрунтованим? A 50-year-old patient who has been suffering from hypertension for more than 10 years had a sudden rise in blood pressure to 200/110 mmHg due to stress. Condition was accompanied by body tremors, headache, tachycardia, general excitement, a feeling of heat and dryness in the mouth. Which drugs are the most justified?

Сечогінні Diuretics

Інгібітори АПФ ACE inhibitors

в-адреноблокатори v-adrenoblockers

Блокатори рецепторів ангіотензину II Angiotensin II receptor blockers

Антагоністи кальцію Calcium antagonists

121 / 200
У пацієнта 54-х років м’яка первинна артеріальна гіпертензія, ІХС: стенокардія напруги II ФК, СН II ст. Супутній діагноз: гастроезофагеальна рефлюксна хвороба, ерозивний езофагіт II стадія. Постійний прийом якого з препаратів може викликати у хворого посилення проявів гастроентерологічної патології? A 54-year-old patient has mild primary arterial hypertension, coronary artery disease: angina pectoris of the II FC, HF of the II stage. Associated diagnosis: gastroesophageal reflux disease, erosive esophagitis Stage II. The constant intake of which of the drugs can cause the patient to increase the manifestations of gastroenterological pathology?

Омепразол Omeprazole

Ьосорбід динітрат Osorbide dinitrate

Еналаприл малеат Enalapril Maleate

Метопролол Metoprolol

Гідрохлортіазид Hydrochlorothiazide

122 / 200
Хвора 32-х років скаржиться на ’’душевний біль”, поганий настрій, відсутність апетиту, безсоння. Зазначені симптоми з’явилися поступово протягом 3-х місяців без видимої причини. У психічному статусі: на питання відповідає тихим голосом, обличчя сумне, міміка скорботна, рухи вповільнені, фон настрою знижений, емоційно пригнічена, темп мислення вповільнений. Стан хворої поліпшується ввечері й погіршується рано вранці. До якого спеціаліста необхідно направити хвору? A 32-year-old patient complains of ``mental pain,'' bad mood, lack of appetite, insomnia. These symptoms appeared gradually over the course of 3 months without of an apparent cause. In the mental status: questions are answered in a low voice, the face is sad, the expression is sad, the movements are slowed down, the background of the mood is lowered, emotionally depressed, the pace of thinking is slowed down. The patient's condition improves in the evening and worsens early in the morning. To which specialist should the patient be referred?

Гастроентеролога Gastroenterologist

Ендокринолога Endocrinologist

Медичного психолога Medical psychologist

Психіатра Psychiatrist

Невролога Neurologist

123 / 200
Хвора 24-х років контактна, правильно орієнтована в місці, часі й власній особистості. Схвильована, не може всидіти на місці. Постійно говорить, що в її ’теперішньому стані винуваті бабки”’ до яких вона ходила; що ’голос у голові повторює, що я неправильно поводжуся, й постійно мучить мене”. Увага звужена в обсязі, прикута до переживань. Критика до стану відсутня. Який з перерахованих препаратів можна призначити пацієнтці в якості основного? A 24-year-old patient is communicative, correctly oriented in place, time and her own personality. She is agitated, cannot sit still. She constantly says that in her 'current the grandmothers' to whom she went were to blame for the condition; that 'the voice in my head repeats that I am behaving incorrectly and constantly torments me'. Attention is narrowed in scope, chained to experiences. There is no criticism of the condition. Which of the listed drugs can be prescribed to a patient in quality of the main?

Бензобарбітал Benzobarbital

Галоперидол Haloperidol

Шіпрамін Shipramine

Препарати літію Lithium drugs

Тразадон Trazadone

124 / 200
Хворому на цукровий діабет I типу, якого доставили до реанімаційного відділення в стані кетоацидотичної коми, в перші три години в якості лікування внутрішньовенно введено: фізіологічний розчин - 2,0 л, розчин Рінгера - 800 мл, розчин калію хлориду 7,5% - 40 мл. Аналіз газів крові показав рН крові, що дорівнює 6,85. Який ще засіб слід ввести хворому? A patient with type I diabetes who was brought to the intensive care unit in a state of ketoacidotic coma was treated intravenously in the first three hours with: physiological solution - 2.0 l, Ringer's solution - 800 ml, potassium chloride solution 7.5% - 40 ml. Blood gas analysis showed a blood pH equal to 6.85. What other means should be administered to the patient?

Плазма крові Blood plasma

Розчин натрію гідрокарбонату Sodium bicarbonate solution

Еритроцитарна маса Erythrocyte mass

Розчин кальцію хлориду Calcium chloride solution

Промедол Promedol

125 / 200
Хворий 34-ти років перебуває на лікуванні в психіатричній лікарні з приводу загострення шизофренії. Об’єктивно: перебуває в ліжку, рухливо загальмований, контакт відсутній. На запитання не відповідає. Поза одноманітна, пацієнт гіпомімічний, наявні симптом ’хоботка”’ воскова гнучкість м’язів, симптом ’повітряної подушки”’ В такому стані лишається близько тижня. Харчування парентеральне. Визначте наявний синдром розладу рухово-вольової сфери: A 34-year-old patient is being treated in a psychiatric hospital due to an exacerbation of schizophrenia. Objectively: he is in bed, he is physically inhibited, there is no contact. When asked, no responds. The posture is monotonous, the patient is hypomimic, the 'proboscis' symptom, waxy flexibility of the muscles, the 'air cushion' symptom are present. He remains in this state for about a week. Parenteral nutrition. Determine the present syndrome of motor-volition disorder:

Апатичний ступор Apathetic stupor

Кататонічний ступор Catatonic stupor

Психогенний ступор Psychogenic stupor

Екзогенний ступор Exogenous stupor

Депресивний ступор Depressive stupor

126 / 200
Хворий 34-х років скаржиться на підвищений апетит, надмірну вагу, задишку під час фізичних навантажень. Під час огляду: надмірне накопичення жиру в ділянці живота та плечового поясу, шкіра блідо-рожева, волосяний покрив на тілі за чоловічим типом, стрій немає, частота серцевих скорочень - 90/хв., артеріальний тиск -120/80 мм рт.ст. Додаткові дослідження: цукор крові - 4,9 ммоль/л, холестерин - 6,2 ммоль/л. Офтальмоскопія: очне дно без змін. Огляд невропатолога: здоровий. Поставте діагноз: A 34-year-old patient complains of increased appetite, overweight, shortness of breath during physical exertion. During the examination: excessive accumulation of fat in the abdomen and shoulder girdle, pale pink skin, male-type body hair, no striae, heart rate - 90/min, blood pressure -120/80 mm Hg. Additional tests: blood sugar - 4.9 mmol/l, cholesterol - 6.2 mmol/l. Ophthalmoscopy: fundus unchanged. Neuropathologist's examination: healthy. Make a diagnosis:

Вторинне ендокринне гіпотиреоїдне ожиріння Secondary endocrine hypothyroid obesity

Первинне алiментарно-конституціональне ожиріння гіноїдний тип Primary alimentary-constitutional obesity gynoid type

Вторинне церебральне ожиріння Secondary cerebral obesity

Вторинне ендокринне гiпооварiальне ожиріння Secondary endocrine hypoovarian obesity

Первинне аліментарно-конституціональне ожиріння, андроїдний тип Primary alimentary-constitutional obesity, android type

127 / 200
Жінка 60-ти років скаржиться на періодичні болі у колінних суглобах, що виникають частіше під час тривалого ходіння, спуску сходами, ввечері, непокоять у першій половині ночі і вщухають до ранку після тривалого відпочинку. В ході обстеження виявлена надмірна маса тіла. Суглоби зовнішньо не змінені, обсяг рухів в них не обмежений. На рентгенограмі правого колінного суглоба видно остеофіти. Для профілактики подальшого прогресування захворювання слід рекомендувати: A 60-year-old woman complains of periodic pains in the knee joints, which occur more often during long walks, going down stairs, in the evening, restlessness in the first half of the night and subside until the morning after a long rest. During the examination, excessive body weight was revealed. The joints are not externally changed, their range of motion is not limited. Osteophytes are visible on the radiograph of the right knee joint. To prevent further progression of the disease, it is recommended:

Регулярний прийом алопуринолу Regular intake of allopurinol

Уникати підвищених навантажень на колінні суглоби Avoid increased loads on knee joints

Щоденний біг підтюпцем Daily jogging

Короткочасна іммобілізація суглоба лонгетою Short-term immobilization of the joint with a splint

Обмежити у дієті продукти, багаті на пурини Limit foods rich in purines in your diet

128 / 200
У повторнороділлі 30-ти років пологи тривають 8 годин. Перейми через кожну хвилину по 50 секунд, активні. Серцебиття плоду - 156/хв., ритмічне. Під час зовнішнього дослідження голівка розташована в порожнині малого тазу. Вагінально: розкриття шийки матки повне, голівка плоду в площині виходу з малого тазу. Стрілоподібний шов в прямому розмірі, мале тім’ячко біля лона. Який це період пологів? In a repeat birth of 30 years, labor lasts 8 hours. Contractions every minute for 50 seconds, active. Fetal heartbeat - 156/min, rhythmic. During external examination, the head is located in the cavity of the small pelvis. Vaginal: opening of the cervix is complete, the fetal head is in the plane of exit from the small pelvis. Arrow-shaped seam in a straight size, small crown near the womb. What is the period of childbirth?

Активна фаза першого періоду нормальних пологів Active phase of the first period of normal childbirth

Другий період нормальних пологів Second period of normal childbirth

Стрімкі пологи Rapid childbirth

Прелімінарний період Preliminary period

Латентна фаза першого періоду нормальних пологів Latent phase of the first period of normal childbirth

129 / 200
Пацієнтка 30-ти років скаржиться на відсутність вагітності протягом 3-х років після заміжжя. Підвищеного харчування, за середньою лінією живота, на внутрішній поверхні стегон і у навколососковій ділянці відзначається ріст волосся. Менструації з 16-ти років, нечасті та нерясні. УЗД: матка звичайних розмірів, яєчники - 4х5х5 см, з великою кількістю кістозних включень. Який найбільш імовірний діагноз? A 30-year-old patient complains of the absence of pregnancy for 3 years after marriage. Increased nutrition, along the midline of the abdomen, on the inner surface of the thighs and in the area around the nipple hair growth is noted. Menstruation since the age of 16, infrequent and scanty. Ultrasound: uterus of normal size, ovaries - 4x5x5 cm, with a large number of cystic inclusions. What is the most likely diagnosis?

Хронічний оофорит Chronic oophoritis

Полікістоз яєчників Polycystic ovary

Двосторонні пухлини яєчників Bilateral ovarian tumors

Кістома яєчників Ovarian Cystoma

Порушення менструального циклу Menstrual cycle disorder

130 / 200
Хворий 38-ми років скаржиться на задишку, відчуття важкості в правому підребер’ї. 2 роки тому переніс туберкульоз легенів. Шийні вени набухли. Пульс - 96/хв., ритмічний, малий, м’який. Тони серця дуже послаблені. Печінка +7 см. Вільна рідина в черевній порожнині. ЕКГ - низький вольтаж, відхилення ЕВС вправо. Рентгенологічно: ”мале серце’.’ Який найбільш імовірний діагноз? A 38-year-old patient complains of shortness of breath, a feeling of heaviness in the right hypochondrium. He suffered from pulmonary tuberculosis 2 years ago. Neck veins are swollen. Pulse - 96/min ., rhythmic, small, soft. Heart sounds are very weak. Liver +7 cm. Free fluid in the abdominal cavity. ECG - low voltage, EBS deviation to the right. X-ray: 'small heart'. What is the most likely diagnosis?

Ексудативний перикардит Exudative pericarditis

Міокардит Myocarditis

Гіпертрофічна кардіоміопатія, обструктивна форма Hypertrophic cardiomyopathy, obstructive form

Констриктивний перикардит Constrictive pericarditis

Дефект міжпередсердної перетинки Atrial membrane defect

131 / 200
Хворий 48-ми років скаржиться на кволість, пітливість, інтенсивне свербіння шкіри, хвилеподібну лихоманку, збільшення шийних і надключичних лімфовузлів. Об’єктивно: блідість шкіри та слизових, шийні лімфовузли - рухомі, щільно-еластичні, розміром до 3 см, не болючі, не спаяні зі шкірою. У крові: еритроцити - 3,0-1012/л, Нb- 100 г/л, лейкоцити -14*109 /л, еоз.- 6%, баз.- 3%, пал.- 11%, сегм.- 69%, лімф.- 7%, мон.- 4%, тромбоцити - 280*109 /л, швидкість осідання еритроцитів - 37 мм/год. Які морфологічні ознаки може бути знайдено при біопсії лімфовузла? A 48-year-old patient complains of weakness, sweating, intense itching of the skin, wave-like fever, enlargement of cervical and supraclavicular lymph nodes. Objectively: pallor of the skin and mucous membranes, cervical lymph nodes - mobile, dense and elastic, up to 3 cm in size, not painful, not fused to the skin. In the blood: erythrocytes - 3.0-1012/l, Hb - 100 g/l, leukocytes -14*109 /l, eoz.- 6%, base.- 3%, pal.- 11%, segm.- 69%, lymph.- 7%, mon.- 4%, platelets - 280*109 /l, erythrocyte sedimentation rate - 37 mm /h. What morphological signs can be found during a biopsy of a lymph node?

Плазмоцити Plasmocytes

Тільця Меллорі Mallory bodies

Клітини Боткіна-Гумпрехта Botkin-Gumprecht cells

Тільця Гейнца Heinz bodies

Клітини Березовського-Штернберга Berezovsky-Sternberg cells

132 / 200
Жінка 72-х років хворіє на цукровий діабет II типу, супутня патологія - гіпертонічна хвороба II ст., серцева недостатність II Б ст. Використовує метформін. Напередодні перенесла гіпертонічний криз після чого з’явилися різка слабкість, міалгії, збільшилася спрага, сухість у роті, поліурія. Артеріальний тиск - 140/95 мм рт.ст., частота серцевих скорочень - 98/хв., набряки та запах ацетону відсутні. Які заходи слід вжити для хворої, щоб попередити розвиток коматозного стану? A 72-year-old woman suffers from type II diabetes, concomitant pathology - hypertension of the II stage, heart failure of the II B stage. She uses metformin. The day before, she had a hypertensive crisis followed by sharp weakness, myalgias, increased thirst, dry mouth, polyuria. Blood pressure - 140/95 mm Hg, heart rate - 98/min, swelling and acetone smell are absent. What measures should be taken take for the patient to prevent the development of a comatose state?

Відміна метформіну, призначення інсуліну короткої дії Withdrawal of metformin, appointment of short-acting insulin

Збільшення дози метформіну в два рази Metformin dose doubled

Додаткове призначення пролонгованого інсуліну Additional assignment of extended-release insulin

Призначення глібенкламіду Prescribing Glibenclamide

Використання гіпотонічного розчину хлориду натрію Use of hypotonic sodium chloride solution

133 / 200
На диспансерному обліку у терапевта поліклініки стоїть чоловік 59-ти років, у якого гіпертонічна хвороба II ст. Хворий регулярно приймає інгібітори АПФ та антагоністи кальцію. З якою кратністю терапевт повинен оглядати хворого (окрім періодів загострення)? A 59-year-old man with stage II hypertension is on the dispensary record of the polyclinic therapist. The patient regularly takes ACE inhibitors and calcium antagonists. How often does the therapist should examine the patient (except during periods of exacerbation)?

1 раз на рік Once a year

1 раз на 9 місяців 1 time in 9 months

1 раз на 4 місяці 1 time in 4 months

1 раз на 6 місяців 1 time in 6 months

1 раз на 3 місяці 1 time in 3 months

134 / 200
Дитина 6-ти років з наявністю анемічного, геморагічного синдромів. В крові Нb- 80 г/л, КП- 0,9, ретикулоцити -20 / 00, лейкоцити - 1,0-109 /л, тромбоцити -10*109 /л. Який діагноз є найбільш імовірним? A 6-year-old child with anemic, hemorrhagic syndromes. Blood Hb - 80 g/l, CP - 0.9, reticulocytes -20 / 00, leukocytes - 1.0-109 /l, platelets -10*109 /l. What diagnosis is the most probable?

Тромбоцитопенічна пурпура Thrombocytopenic purpura

Апластична анемія Aplastic anemia

Дефіцитна анемія Deficiency anemia

Тромбоцитопатія Thrombocytopathy

Лімфобластний лейкоз Lymphoblastic leukemia

135 / 200
Під час судово-медичного дослідження трупа судмедексперт описав у тім’яно-скроневій ділянці справа рану лінійної форми, розміром - 6,4 см при зведених краях, краї нерівні, вкриті саднами; в глибині рани видно тканинні перетинки. Дайте назву описаної рани: During the forensic medical examination of the corpse, the forensic medical expert described a wound of a linear shape on the right in the parietal-temporal area, the size of which is 6.4 cm with folded edges, the edges are uneven , covered with scabs; tissue membranes are visible in the depth of the wound. Give the name of the described wound:

Рубана Rubana

Різана Rizana

Розсічена Dissected

Забита Bulk

Колота Kolata

136 / 200
На вулиці знайдено труп жінки 24-х років з імовірним отруєнням. Після огляду місця події та трупа слідчий призначив судово-медичне дослідження. Відповідно до чинного Кримінально-процесуального кодексу України призначення такого дослідження в цьому випадку є обов’язковим для: The corpse of a 24-year-old woman was found on the street with suspected poisoning. After examining the scene and the corpse, the investigator ordered a forensic examination. In accordance with the current Criminal Procedure Code of Ukraine, the appointment of such a study in this case is mandatory for:

Визначення причини смерті Determining the cause of death

Визначення механізму настання смерті Determining the mechanism of death

Визначення давнини настання смерті Determining the age of death

Визначення виду смерті Definition of type of death

Визначення роду смерті Definition of type of death

137 / 200
Пацієнт 60-ти років скаржиться на практично постійне відчуття важкості та переповнення в епігастрії, що посилюється після їжі, відрижку з тухлим запахом, іноді блюванні з’їденою 1-2 дні тому їжею, схуднення. 12 років тому у нього була вперше виявлена виразка пілоричного каналу. Відзначав періодичні ’’голодні” болі, з приводу яких приймав омепразол. Погіршення протягом 3-х місяців. Об’єктивно: визначається ”шум плескоту” в епігастрії. Про яке ускладнення йдеться? A 60-year-old patient complains of an almost constant feeling of heaviness and fullness in the epigastrium, which worsens after eating, belching with a rotten smell, sometimes vomiting eaten 1- 2 days ago I was eating, losing weight. 12 years ago, he was first diagnosed with a pyloric canal ulcer. He noted periodic 'hunger' pains, for which he took omeprazole. Worsening over the course of 3 months. Objectively: a 'splashing noise' is detected in epigastrium. What complication is it about?

Пенетрація виразки шлунка Stomach ulcer penetration

Стеноз пілоруса Stenosis of the pylorus

Стороннє тіло шлунка (безоар) Foreign body of the stomach (bezoar)

Малігнізація виразки шлунка Malignancy of gastric ulcer

Функціональний спазм воротаря Functional spasm of the goalkeeper

138 / 200
Пацієнтка 40-ка років хворіє на фіброміому матки. Протягом останніх 10 років зазначає тривалі рясні менструації. Скарги на слабкість, серцебиття, задишку під час фізичного навантаження, зниження працездатності. Об’єктивно: шкіра бліда, суха, спостерігаються ламкість нігтів, зниження сили м’язів, атрофія сосочків язика. Частота серцевих скорочень - 96/хв. Систолічний шум на верхівці серця. Еритроцити -2,6-1012/л, НЬ- 70 г/л, кП- 0,7, ретикулоцити - 1,2, лейкоцити - 4,6-109 /л, тромбоцити - 170*109 /л. Яке захворювання може бути у пацієнтки? A 40-year-old female patient suffers from uterine fibroids. For the past 10 years, she has noted prolonged heavy menstruation. Complaints of weakness, palpitations, shortness of breath during physical exertion, reduced work capacity . Objectively: the skin is pale, dry, brittle nails, decreased muscle strength, atrophy of tongue papillae are observed. Heart rate - 96/min. Systolic murmur at the top of the heart. Erythrocytes - 2.6-1012/l, НБ- 70 g/l, kP - 0.7, reticulocytes - 1.2, leukocytes - 4.6-109 /l, platelets - 170*109 /l. What disease can the patient have?

Гостра анемія Acute anemia

Гіпопластична анемія Hypoplastic anemia

Хронічна постгеморагічна анемія Chronic posthemorrhagic anemia

Таласемія Thalassemia

Залізодефіцитна анемія Iron deficiency anemia

139 / 200
Хворий 39-ти років скаржиться на запаморочення, швидку втомлюваність. Три доби тому прооперований з приводу активної виразки дванадцятипалої кишки, ускладненої кровотечею. Операція - вшивання кровоточивої виразки. Об’єктивно: шкірні покриви бліді. Артеріальний тиск - 100/60 мм рт.ст., пульс - 98/хв. В крові: еритроцити - 2,8-1012/л, Hb- 76 г/л, КП- 0,8, ретикулоцити - 8%, тромбоцити - 320*109 /л, лейкоцити - 9,0 Г/л, швидкість осідання еритроцитів - 20 мм/год. Який найбільш імовірний діагноз? A 39-year-old patient complains of dizziness, rapid fatigue. Three days ago, he was operated on for an active duodenal ulcer complicated by bleeding. The operation was suturing a bleeding ulcer. About Objectively: the skin is pale. Blood pressure - 100/60 mm Hg, pulse - 98/min. In the blood: erythrocytes - 2.8-1012/l, Hb - 76 g/l, CP - 0.8 , reticulocytes - 8%, platelets - 320*109 /l, leukocytes - 9.0 G/l, erythrocyte sedimentation rate - 20 mm/h. What is the most likely diagnosis?

Гіпопластична анемія Hypoplastic anemia

Нейроциркуляторна дистонія Neurocirculatory dystonia

B12-дефіцитна анемія B12-deficiency anemia

Гостра постгеморагічна анемія Acute posthemorrhagic anemia

Гемолітична анемія Hemolytic anemia

140 / 200
У електрозварювальника зі стажем роботи 15 років під час медичного огляду виявлено сухі хрипи в нижніх відділах легень. На рентгенограмі спостерігаються дифузні вузлики розміром 3-4 мм в середніх і нижніх відділах легень. Яке захворювання можна припустити? During a medical examination, an electric welder with 15 years of experience was found to have dry wheezing in the lower parts of the lungs. On the radiograph, diffuse nodules of 3-4 mm in size are observed in the middle and lower lung departments. What disease can be assumed?

Карбоконіоз Carboconiosis

Силікатоз Silicatosis

Металоконіоз Metaloconiosis

Силікоз Silicosis

Бронхіт Bronchitis

141 / 200
Чоловік 65-ти років скаржиться на задишку інспіраторного характеру. Хворіє на КС 15 років. Артеріальний тиск -150/90 мм рт.ст., частота серцевих скорочень - 52/хв. Тони серця глухі, акцент II тону над легеневою артерією. У легенях: дрібноміхурцеві хрипи у нижніх відділах. ЕКГ: патологічний Q у III, aVF відвіденнях без динаміки. ЕхоКГ: дилатація лівих відділів, фракція викиду лівого шлуночка -35%. Що лежить у основі гемодінамічних розладів у хворого? A 65-year-old man complains of shortness of breath of an inspiratory nature. He has been suffering from CS for 15 years. Blood pressure -150/90 mm Hg, heart rate - 52/min. Heart sounds are dull, emphasis of the II tone over the pulmonary artery. In the lungs: small alveolar rales in the lower parts. ECG: pathological Q in III, aVF visits without dynamics. Echocardiogram: dilatation of the left parts, ejection fraction of the left ventricle -35%. What is the basis of the patient's hemodynamic disorders?

Метаболічна дисфункція Metabolic dysfunction

Діастолічна дисфункція Diastolic dysfunction

Систолічна дисфункція Systolic dysfunction

Систоло-діастолічна дисфункція Systolic-diastolic dysfunction

Порушення периферичного опору Violation of peripheral resistance

142 / 200
Хворий 62-х років випадково намацав у себе в лівій підключичній ямці щільний, розміром трохи більший за горошину, рухливий, не спаяний зі шкірою вузлик. При розпитуванні з’ясувалося, що за останні 6 місяців він схуд на 12 кг. Зазначає слабкість, зменшення працездатності, зниження апетиту. Яке дослідження є першочерговим для встановлення діагнозу? A 62-year-old patient accidentally felt in his left subclavian fossa a dense, slightly larger than a pea-sized, mobile nodule not fused to the skin. it became clear that over the past 6 months he has lost 12 kg. He notes weakness, reduced work capacity, decreased appetite. What research is the first priority for establishing a diagnosis?

Стернальна пункція Sternal puncture

Пункція лімфатичного вузла Lymph node puncture

ЕФГДС EFGDS

УЗД органів черевної порожнини Ultrasound of abdominal organs

Рентгенографія органів грудної клітки X-ray of chest organs

143 / 200
Хворий 25-ти років спостерігається у гематолога з приводу гемофілії А. Після падіння з турніка виник гострий гемартроз колінного суглоба. Об’єктивно: правий колінний суглоб різко збільшений в об’ємі, шкіра над ним гіперемована. Час кровотечі за Дьюком - 3 хвилини, час осідання крові за Лі-Уайтом - 20 хвилин. Який препарат найбільш ефективний для лікування даного хворого? A 25-year-old patient is seen by a hematologist for hemophilia A. Acute hemarthrosis of the knee joint occurred after falling from a horizontal bar. Objectively: the right knee joint is sharply enlarged in volumes, the skin over it is hyperemic. Duke's bleeding time is 3 minutes, Lee-White's blood settling time is 20 minutes. Which drug is the most effective for the treatment of this patient?

Рекомбінантний Х фактор Recombinant X factor

Амбен Amben

Рекомбінантний VIII фактор Recombinant factor VIII

Тромбоконцентрат Thromboconcentrate

Амінокапронова кислота Aminocaproic acid

144 / 200
Хвора захворіла гостро, через 2 дні після повернення із поїздки до Нігерії. Скарги на часті рідкі випорожнення у вигляді ”рисового відвару”’ нестримне блювання, судоми в м’язах. Об’єктивно: температура тіла - 35,0oC, пульс - 120/хв., артеріальний тиск - 50/30 мм рт.ст. Риси обличчя загострені, темні кола під очима. Живіт безболісний. Який діагноз найбільш ймовірний? The patient became acutely ill, 2 days after returning from a trip to Nigeria. Complaints of frequent liquid stools in the form of 'rice broth'' uncontrollable vomiting, muscle cramps' ulcers. Objectively: body temperature - 35.0oC, pulse - 120/min., blood pressure - 50/30 mm Hg. Facial features are sharpened, dark circles under the eyes. The abdomen is painless. What is the most likely diagnosis?

Геморагічна гарячка Hemorrhagic fever

Холера Cholera

Сальмонельоз Salmonellosis

Малярія Malaria

Харчова токсикоінфекція Food poisoning

145 / 200
Після ускладнених пологів у немовляти виник параліч правої руки, м’язовий тонус знижений, рефлекси не викликаються. Яке ускладнення виникло у немовляти? After a complicated delivery, the baby developed paralysis of the right hand, muscle tone is reduced, reflexes are not triggered. What complication did the baby have?

Плексит правого плечового сплетіння Plexitis of the right brachial plexus

Дитячий церебральний параліч Cerebral palsy

Вроджена вада розвитку Congenital malformation

Шийна радикулопатія Cervical radiculopathy

Сирингомієлія Syringomyelia

146 / 200
Хвора 48-ми років скаржиться на біль та слабкість у м’язах шиї, верхніх та нижніх кінцівок, лихоманку. Хворіє близько 2-х років. Об’єктивно: температура тіла -37,8°C. Периорбітально бузково-рожевий колір шкіри. При пальпації м’язів верхніх та нижніх кінцівок болючість та ущільнення. Зниження сили м’язів кінцівок. У крові швидкість осідання еритроцитів - 45 мм/год. Для верифікації діагнозу найбільш доцільно виконати: A 48-year-old patient complains of pain and weakness in the muscles of the neck, upper and lower limbs, fever. She has been ill for about 2 years. Objectively : body temperature -37.8°C. Periorbital lilac-pink color of the skin. When palpating the muscles of the upper and lower limbs, soreness and tightness. Decreased strength of the muscles of the limbs. In the blood, the sedimentation rate of erythrocytes is 45 mm/h. For verification it is most expedient to carry out the diagnosis:

Визначення рівня креатинфосфокінази крові Determination of blood creatine phosphokinase

Дослідження рівню імуноглобулінів Investigation of the level of immunoglobulins

Дослідження мікроциркуляції Research of microcirculation

Дослідження біоптату м’язів Research of muscle biopsy

Визначення антитіл до нативної ДНК Definition of antibodies to native DNA

147 / 200
Хлопчику 12 років. Протягом року у дитини періодично виникають напади болю в навколопупковій ділянці та епігастрії, нудота, зниження апетиту, метеоризм, рідкі випорожнення. Пальпаторно: болючість в епігастрії, панкреатичній точці Дежардена, холедохопанкреатичній зоні Шоффара, позитивний симптом Мейо-Робсона. Рівень амілази крови та сечі підвищений, хлориди поту - 17 ммоль/л. УЗД підшлункової залози: підвищення ехогенності, розширення панкреатичної протоки. Яким буде діагноз? The boy is 12 years old. During the year, the child periodically has attacks of pain in the peri-umbilical region and epigastrium, nausea, decreased appetite, flatulence, loose stools. Palpation: pain in the epigastrium , Desjardin's pancreatic point, Shofar's choledochopancreatic zone, Mayo-Robson's positive sign. Blood and urine amylase levels are elevated, sweat chlorides are 17 mmol/l. Pancreas ultrasound: increased echogenicity, pancreatic duct dilation. What will be the diagnosis?

Хронічний холецистит Chronic cholecystitis

Функціональна діарея Functional diarrhea

Хронічний панкреатит Chronic pancreatitis

Муковісцидоз Cystic Fibrosis

Виразкова хвороба дванадцятипалої кишки Duodenal ulcer disease

148 / 200
Хворий 68-ми років звернувся до урологічної клініки зі скаргами на поклики до сечопуску, відсутність сечі протягом останніх 10 годин. Об’єктивно: над лобком пальпується болючий утвір, верхній край якого сягає пупка, в ході ректального дослідження передміхурова залоза збільшена, безболісна, щільної консистенції. Додаткове дослідження виявило рівень простатспецифічного антигену в крові - 3,2 нг/мл. Який найбільш імовірний діагноз? A 68-year-old patient came to the urology clinic with complaints of urges to urinate, no urine for the past 10 hours. Objectively: a painful lump is palpated above the pubis, the upper edge of which reaches the navel, during a rectal examination, the prostate gland is enlarged, painless, with a dense consistency. An additional examination revealed the level of prostate-specific antigen in the blood - 3.2 ng/ml. What is the most likely diagnosis?

Доброякісна гіперплазія передміхурової залози, гостра затримка сєчі Benign prostatic hyperplasia, acute urinary retention

Склероз передміхурової залози, гостра затримка сечі Sclerosis of the prostate, acute urinary retention

Рак передміхурової залози, гостра затримка сечі Prostate cancer, acute urinary retention

Доброякісна гіперплазія передміхурової залози, хронічна затримка сечі Benign prostatic hyperplasia, chronic urinary retention

Хронічний парапроктит, гостра затримка сечі Chronic paraproctitis, acute urinary retention

149 / 200
Хвора на цукровий діабет 64-х років лікувалася метформіном. У зв’язку з підвищеним артеріальним тиском приймала сечогінні. Поступово ефективність діурети-ка знизилась, з’явилися нудота, блювання. Контакт з хворою утруднений. Шкіра суха. Запах ацетону відсутній. Артеріальний тиск - 180/100 мм рт.ст. Тони серця глухі. Пульс - 98/хв. Дихання везикулярне. Живіт болючий в епігастрії. Печінка +4 см. Глюкоза крові - 48 ммоль/л, Na156 ммоль/л, K- 5,2 ммоль/л, сечовина - 15 ммоль/л. З якого фармакологічного підходу слід починати? A 64-year-old diabetic patient was treated with metformin. Due to high blood pressure, she took diuretics. Gradually, the effectiveness of the diuretic decreased, nausea appeared , vomiting. Contact with the patient is difficult. The skin is dry. The smell of acetone is absent. Blood pressure - 180/100 mm Hg. Heart sounds are dull. Pulse - 98/min. Vesicular breathing. Abdominal pain in the epigastrium. Liver +4 cm. Glucose blood - 48 mmol/l, Na156 mmol/l, K - 5.2 mmol/l, urea - 15 mmol/l. What pharmacological approach should be started?

Перевести хвору на лікування протамін-цинк інсуліном Transfer the patient to treatment with protamine-zinc insulin

Невідкладна гідратація хворої Urgent hydration of the patient

Корекція вмісту Na+ у крові Correction of Na+ content in blood

Призначення бікарбонату натрію Purpose of sodium bicarbonate

Корекція артеріального тиску за допомогою сульфату натрію Correction of blood pressure with sodium sulfate

150 / 200
Дівчинка 14-ти років під час забору крові втратила свідомість. Напередодні скаржилася на головний біль. Шкіра бліда. Частота дихання - 20/хв., частота серцевих скорочень - 80/хв., артеріальний тиск -90/60 мм рт.ст. Живіт м’який. Менінгеальні симптоми відсутні. Який попередній діагноз? A 14-year-old girl lost consciousness during blood sampling. The day before, she complained of a headache. The skin is pale. Breathing rate - 20/min., heart rate - 80/min., blood pressure -90/60 mm Hg. The abdomen is soft. There are no meningeal symptoms. What is the previous diagnosis?

Гостра лівошлуночкова недостатність Acute left ventricular failure

Колапс Collapse

Епілепсія Epilepsy

Дихальна недостатність Respiratory failure

Непритомність Fainting

151 / 200
-ти років скаржиться на підвищення температури тіла до 41o 151. C, головний біль, слабкість. Захворів гостро, на 5-й день захворювання з’явився розеолезно-петехіальний висип на бічній поверхні грудної клітки, спини. РЗК з рикетсіями Провачека - 1:640, lgM89%. Який найбільш імовірний діагноз? year-old complains of an increase in body temperature up to 41o 151.C, headache, weakness. He became acutely ill, on the 5th day of the illness, roseolosis appeared a petechial rash on the lateral surface of the chest, back. RZK with rickettsia Provacek - 1:640, lgM89%. What is the most likely diagnosis?

Хвороба Брілла Brill's disease

Ентеровірусна інфекція Enterovirus infection

Висипний тиф Typhoid

Грип Flu

Черевний тиф Typhoid

152 / 200
У хворого 26-ти років півгодини тому після прийому амінопеніциліну з приводу гострого бронхіту з’явилися задишка мішаного характеру, сухий кашель, свербіння шкіри. Об’єктивно: набряк та почервоніння шкіри в ділянці орбіти, дихання свистяче, пульс - 114/хв., артеріальний тиск - 90/60 мм рт.ст. Препаратом невідкладної допомоги буде: Half an hour ago, after taking aminopenicillin for acute bronchitis, a 26-year-old patient developed dyspnea of a mixed nature, a dry cough, and itchy skin. Objectively: swelling and redness of the skin in the orbital area, wheezing, pulse - 114/min., blood pressure - 90/60 mm Hg. The emergency medicine will be:

Димедрол Diphenhydramine

Активоване вугілля Activated carbon

Гепарин Heparin

Еуфілін Euphilin

Преднізолон Prednisone

153 / 200
У хворого 18-ти років артеріальний тиск -120/70 мм рт.ст. В III міжребер’ї зліва від краю грудини систолічне тремтіння. Ліва межа серця зміщена на 1 см назовні. У II-III міжребер’ях зліва систоло-діастолічний шум, який посилюється під час систоли, акцент II тону. Який діагноз є найбільш імовірним? An 18-year-old patient has a blood pressure of -120/70 mm Hg. In the III intercostal space to the left of the edge of the sternum, there is a systolic tremor. The left border of the heart is displaced 1 cm outward. In the II-III intercostal space on the left, systolic-diastolic murmur, which increases during systole, the accent of the II tone. What diagnosis is the most probable?

Дефект міжшлуночкової перегородки Ventricular septal defect

Коарктація аорти Coarctation of the aorta

Відкрита артеріальна протока Open ductus arteriosus

- -

Дефект міжпередсердної перегородки Atrial septal defect

154 / 200
через 3 тижні після пологів підвищилася температура тіла до 38o 154. C, з’явилися остуда, слабкість та біль в лівій молочній залозі. Молочна залоза збільшена, нагрубла, болюча при пальпації; розм’якшення та флуктуації в ділянці інфільтрату немає. В крові - помірний лейкоцитоз. Який найбільш імовірний діагноз? 3 weeks after childbirth, the body temperature rose to 38o 154. C, chills, weakness and pain appeared in the left mammary gland. The mammary gland was enlarged, rough, painful on palpation; there is no softening and fluctuation in the area of the infiltrate. There is moderate leukocytosis in the blood. What is the most likely diagnosis?

Абсцедивний мастит Abscessive mastitis

Серозний мастит Serous mastitis

Мастопатія Mastopathy

Гангренозний мастит Gangrenous mastitis

Лактостаз Lactostasis

155 / 200
У жінки 28-ми років на 7-му добу після пологів з’явилися скарги на біль у правій молочній залозі, підвищення температури тіла до 39oC, остуда, погіршення загального стану. Під час огляду визначається інфільтрат у верхньозовнішньому квадранті молочної залози з ділянкою розм’якшення. Якою буде тактика лікаря? On the 7th day after childbirth, a 28-year-old woman complained of pain in the right mammary gland, an increase in body temperature to 39oC, chills, worsening general condition. During the examination, an infiltrate is detected in the upper outer quadrant of the mammary gland with an area of softening. What will the doctor's tactics be?

Антибактеріальна терапія Antibacterial therapy

Припинення лактації Stop lactation

Хірургічне лікування Surgical treatment

Динамічне спостереження Dynamic Observation

Фізіотерапевтичне лікування Physiotherapy treatment

156 / 200
В місті N проводилося вивчення захворюваності на інфаркт міокарда за попередні роки (з 2009 по 2013 роки). Який вид епідеміологічного дослідження був використаний? In the city N, the incidence of myocardial infarction was studied in previous years (from 2009 to 2013). What type of epidemiological study was used?

Аналітичний Analytic

Проспективний Prospective

Описовий Descriptive

Ретроспективний Retrospective

Експериментальний Experimental

157 / 200
Хвора 52-х років впродовж 2-х років спостерiгає збільшення правої молочної залози. Протягом останніх 3-х місяців з’явилося почервоніння шкіри цієї молочної залози. У правій аксілярній ділянці пальпується туго-еластичної консистенції лімфовузол розміром до 1,5 см. Права молочна залоза збільшена у розмірах, шкіра її гіперемована, симптом лимонної шкірки, сосок втягнутий. Яке захворювання найбільш імовірне у даному випадку? A 52-year-old patient has been observing an increase in her right breast for 2 years. Over the past 3 months, redness of the skin of this breast has appeared. In the right in the axillary area, a lymph node of a tight-elastic consistency up to 1.5 cm in size is palpated. The right mammary gland is enlarged, its skin is hyperemic, the symptom of lemon peel, the nipple is retracted. What disease is most likely in this case?

Мастит Mastitis

Рак молочної залози Breast cancer

Пахвовий лімфаденіт Axillary lymphadenitis

Фіброаденома молочної залози Breast fibroadenoma

Мастопатія Mastopathy

158 / 200
У хворої 49-ти років скарги на стискання у ділянці серця, слабкість, акроціаноз, розширення підшкірних вен, асцит, набряки ніг, пульсацію вен шиї. ЦВТ- 200 мм вод.ст., пульс слабкого наповнення -100/хв., тони серця приглушені, шуму немає. Рентгенологічно: маленьке із чіткими контурами серце, вапняні відкладення у порожнині перикарду. Який найбільш імовірний діагноз? A 49-year-old patient complains of compression in the heart area, weakness, acrocyanosis, dilatation of subcutaneous veins, ascites, swelling of the legs, pulsation of the neck veins. CVT-200 mm H2O, weak heart rate -100/min, muffled heart sounds, no noise. X-ray: small heart with clear contours, calcareous deposits in the pericardial cavity. What is the most likely diagnosis?

Випітний перикардит Effluent pericarditis

Лівобічна пневмонія Left-sided pneumonia

Мітральний стеноз Mitral stenosis

Міокардит Myocarditis

Стискальний (констриктивний) перикардит Constrictive pericarditis

159 / 200
Хвора 52-х років скаржиться на періодичний нападоподібний біль у правій поперековій ділянці, який віддає у пахвинну ділянку і внутрішню поверхню правого стегна, порушення сечовиділення, яке супроводжується різями і домішками крові у сечі. У сечі: питома вага -1014, білок - 0,078 г/л, еритроцити - 1/2 поля зору, лейкоцити - 5-7 в полі зору, велика кількість кристалів сечової кислоти. Вкажіть найбільш імовірний попередній діагноз: A 52-year-old patient complains of periodic attack-like pain in the right lumbar area, which radiates to the inguinal area and the inner surface of the right thigh, urinary incontinence, which is accompanied by cutting and impurities of blood in the urine. In the urine: specific gravity -1014, protein - 0.078 g/l, erythrocytes - 1/2 field of vision, leukocytes - 5-7 in the field of vision, a large number of uric acid crystals. Specify the most likely preliminary diagnosis:

Гострий цистит Acute cystitis

Новоутворення нирки Kidney Neoplasm

Гострий гломерулонефрит Acute glomerulonephritis

Сечокам’яна хвороба Urolithiasis

Хронічний пієлонефрит Chronic pyelonephritis

160 / 200
Пацієнтка 20-ти років звернулася до лікаря жіночої консультації зі скаргами на відсутність менструацій протягом 7-ми місяців. З анамнезу: в ранньому віці хворіла на дитячі інфекції й ангіни, менархе з 13-ти років, місячні регулярні, менструальний цикл 28 днів, менструація триває 5-6 днів, безболісна. 7 місяців тому перенесла стрес. В ході гінекологічного огляду змін з боку матки і додатків не виявлено. Який найбільш імовірний діагноз? A 20-year-old patient turned to the doctor of the women's consultation with complaints about the absence of menstruation for 7 months. From the anamnesis: she suffered from childhood infections and sore throats at an early age , menarche since the age of 13, periods are regular, the menstrual cycle is 28 days, menstruation lasts 5-6 days, painless. 7 months ago, she experienced stress. During a gynecological examination, no changes were found in the uterus and appendages. What is the most likely diagnosis?

Первинна аменорея Primary amenorrhea

Несправжня аменорея False amenorrhea

Вторинна аменорея Secondary amenorrhea

Альгодисменорея Algodysmenorrhoea

Олігоменорея Oligomenorrhea

161 / 200
Дитині 8 років, скарг немає, активна. Загальний стан не порушений. Помірний систолічний шум з максимальним звучанням в III міжреберному проміжку зліва від грудини, який не змінюється при диханні, роздвоєння другого тону. Який діагноз найбільш імовірний? The child is 8 years old, there are no complaints, he is active. The general condition is not disturbed. A moderate systolic murmur with a maximum sound in the III intercostal space to the left of the sternum, which does not change during breathing , bifurcation of the second tone. What is the most likely diagnosis?

Недостатність трикуспідального клапану Tricuspid valve insufficiency

Пролапс мітрального клапану Mitral valve prolapse

Відкрита артеріальна протока Open ductus arteriosus

Дефект міжпередсердної перегородки Atrial septal defect

Дефект міжшлуночкової перегородки Ventricular septal defect

162 / 200
В ході профогляду встановлено: у жінки 23-х років вагітностей не було. При бімануальному обстеженні виявлено: тіло матки нормальних розмірів, на передній стінці - щільне округле утворення на ніжці, пов’язане з маткою, розмірами в діаметрі до 6-ти см, неболюче, додатки без особливостей. За допомогою УЗД підтверджено діагноз субсерозної міоми матки. Який метод лікування слід запропонувати? During the professional examination, it was established that a 23-year-old woman had no pregnancies. Bimanual examination revealed: the body of the uterus is of normal size, on the front wall - a dense rounded formation on legs, connected to the uterus, up to 6 cm in diameter, painless, appendages without features. The diagnosis of subserous uterine myoma was confirmed by ultrasound. What method of treatment should be offered?

Дефундація матки Defundation of the uterus

Висока надпіхвова ампутація матки High supravaginal amputation of the uterus

Ампутація матки Uterine amputation

Консервативна міомектомія Conservative myomectomy

Екстирпація матки Uterus extirpation

163 / 200
Сімейний лікар протягом останніх трьох років спостерігає зростання захворюваності на дифтерію серед населення, яке він обслуговує. Які лікувально-профілактичні заходи необхідно провести, щоб зменшити захворюваність дифтерією на дільниці? For the past three years, a family doctor has observed an increase in the incidence of diphtheria among the population he serves. What preventive measures should be taken to reduce the incidence of diphtheria in the ward?'

Виявити носіїв і провести для них курс лікування Identify carriers and provide them with a course of treatment

Виконати дизенфекцію в осередку та облік контактних осіб Perform disinfection in the cell and accounting of contact persons

Виконати щеплення населення проти дифтерії Vaccinate the population against diphtheria

Виконати госпіталізацію хворих на дифтерію Hospitalization of patients with diphtheria

Провести біцилінопрофілактику на дільниці Conduct bicillin prophylaxis at the site

164 / 200
Дитині 1 рік, госпіталізована на 4-й день хвороби зі скаргами на підвищення температури тіла до 39oC, задишку. Об’єктивно: ціаноз носогубного трикутника, при аускультації легень справа дихання різко ослаблене, вологі крепітуючі хрипи. Стандартна антибактеріальна терапія впродовж перших трьох днів лікування неефективна. Припущено стафілококову етіологію пневмонії. Яка антибактерiальна терапія найдоцільніша у даному випадку? The child is 1 year old, hospitalized on the 4th day of illness with complaints of an increase in body temperature to 39oC, shortness of breath. Objectively: cyanosis of the nasolabial triangle, on auscultation of the lungs on the right, breathing is sharply weakened, moist crepitus wheezes. Standard antibacterial therapy during the first three days of treatment is ineffective. Staphylococcal etiology of pneumonia is assumed. What antibacterial therapy is most appropriate in this case?

Азитроміцин Azithromycin

Еритроміцин Erythromycin

Пєніцилін Penicillin

Ампіцилін Ampicillin

Ванкоміцин Vancomycin

165 / 200
Хлопчик 10-ти років надійшов до відділення політравми після тупої травми грудної клітки в результаті падіння з велосипеда. При надходженні артеріальний тиск - 110/80 мм рт.ст., пульс - 96/хв. Рентгенограма грудної клітки необтяжена. На ехо-кардіограмі виявлена вільна рідина у порожнині перикарду, в кількості до 100 мілілітрів. Через годину після надходження почали наростати ознаки серцевої недостатності: набрякання шийних вен, зниження артеріального тиску до 90/70 мм рт.ст., прискорення пульсу до 120/хв. Аускультативно серцеві тони ледве чутні. Якою повинна бути першочергова лікарська тактика? A 10-year-old boy was admitted to the polytrauma department after a blunt chest injury as a result of a fall from a bicycle. On admission, blood pressure was 110/80 mm Hg. , pulse - 96/min. The X-ray of the chest is unweighted. The echocardiogram revealed free fluid in the pericardial cavity, in the amount of up to 100 milliliters. An hour after admission, signs of heart failure began to increase: swelling of the neck veins, a drop in blood pressure to 90/70 mm Hg, pulse acceleration up to 120/min. Heart sounds are barely audible on auscultation. What should be the primary medical tactics?

Пункція перикарду Pericard puncture

Діуретики в/в IV diuretics

Антибіотики в/в IV antibiotics

Серцеві глікозиди в/в Cardiac glycosides IV

Постійна оксигенотерапія Continuous oxygen therapy

166 / 200
Жінка 38-ми років скаржиться на сверблячий висип на шкірі, який з’явився через день після прийому анальгетика. Об’єктивно: загальний стан хворої середньої тяжкості, на шкірі обличчя, тулуба й кінцівок рясний уртикарний висип, який місцями зливається. Який лікарський засіб слід призначити хворій в першу чергу? A 38-year-old woman complains of an itchy skin rash that appeared a day after taking an analgesic. Objectively: the general condition of the patient is of moderate severity, on the skin of the face, trunk and limbs has a profuse urticarial rash, which merges in places. What medicine should be prescribed to the patient first of all?

Реосорбілакт Reosorbilact

Розчин хлористого кальцію Calcium chloride solution

Преднізолон Prednisone

Ентеросгель Enterosgel

Тавегіл Tavegil

167 / 200
Хвора 35-ти років скаржиться на виражену загальну слабкість, пожовтіння склер, важкість у правому підребер’ї, періодичну лихоманку, біль в суглобах, кровоточивість ясен. В крові: АлАТ- 1,98 ммоль/гл, АсАТ- 2,5 ммоль/гл, загальний білірубін - 105 мкмоль/л, прямий - 65 мкмоль/л, ANA у титрі 1:160, антитіла до гладеньких м’язів у титрі 1:40, збільшення IgG у 2 рази. НВУ-ДНК(-), НСУ-РНК(-). Який попередній діагноз? A 35-year-old patient complains of pronounced general weakness, yellowing of the sclera, heaviness in the right hypochondrium, periodic fever, pain in the joints, bleeding gums. In the blood : AlAT - 1.98 mmol/hl, AsAT - 2.5 mmol/hl, total bilirubin - 105 μmol/l, direct - 65 μmol/l, ANA at a titer of 1:160, antibodies to smooth muscles at a titer of 1 :40, 2-fold increase in IgG. NSU-DNA(-), NSU-RNA(-). What is the preliminary diagnosis?

Хвороба Жильбера Gilbert's disease

Хвороба Вільсона-Коновалова Wilson-Konovalov disease

Системний червоний вовчак Systemic lupus erythematosus

Аутоімунний гепатит Autoimmune hepatitis

Вірусний гепатит В Viral hepatitis B

168 / 200
Жінка 45-ти років скаржиться на коль-коподібний біль у правому підребер’ї, що виникає після вживання жирної їжі, фізичного навантаження, іррадіює в праву лопатку та праве плече, зменшується при прийомі спазмолітиків. Хворіє впродовж року, періодично зазначає пожовтіння склер під час нападів болю. Який метод обстеження слід призначити в першу чергу для встановлення діагнозу? A 45-year-old woman complains of colic-like pain in the right hypochondrium, which occurs after eating fatty food, physical exertion, radiating to the right shoulder blade and right shoulder, decreases when taking antispasmodics. He has been sick for a year, periodically notes yellowing of the sclera during pain attacks. What examination method should be prescribed first of all to establish a diagnosis?

Комп’ютерну томографію Computed tomography

Дуоденальне зондування Duodenal sounding

Рентгенографію органів черевної порожнини X-ray of abdominal organs

Фіброгастродуоденоскопію Fibrogastroduodenoscopy

Ультрасонографію Ultrasonography

169 / 200
Потерпілий 40-ка років зазнав ножового поранення під праву лопатку. Об’єктивно: шкіра звичайного кольору, пульс - 96/хв., ритмічний, дихання везикулярне з обох боків, частота дихання - 20/хв., артеріальний тиск -130/90 мм рт.ст. На спині рана - 4,0х0,5 см, кровить. На оглядовій рентгенограмі патології не виявлено. Якою буде тактика? A 40-year-old victim was stabbed under the right shoulder blade. Objectively: normal-colored skin, pulse - 96/min., rhythmic, vesicular breathing from both sides, respiratory rate - 20/min., blood pressure -130/90 mm Hg. On the back, a wound - 4.0x0.5 cm, bleeding. No pathology was detected on the X-ray examination. What will be the tactics?

Первинна хірургічна обробка рани Primary surgical treatment of the wound

Торакотомія Thoracotomy

Пункція плевральної порожнини Puncture of the pleural cavity

Торакоскопія Thoracoscopy

Торакоцентез Thoracentesis

170 / 200
Хвора 35-ти років скаржиться на нездужання, лихоманку, біль у горлі та в ділянці шиї з іррадіацією в нижню щелепу. Все це протягом 7 днів супроводжувалося підвищеною знервованістю, пітливістю, втратою ваги. Щитоподібна залоза помірно збільшена і обмежено болюча під час пальпації. В ході обстеження виявлені дані, що відповідають тиреотоксикозу. В крові швидкість осідання еритроцитів - 64 мм/год. Діагностовано підгострий тиреоїдит. Причиною тиреотоксикозу при підгострому тиреоїдиті є: A 35-year-old patient complains of malaise, fever, pain in the throat and neck with radiation to the lower jaw. All this was accompanied by increased nervousness for 7 days, sweating, weight loss. The thyroid gland is moderately enlarged and limitedly painful during palpation. During the examination, data corresponding to thyrotoxicosis were found. In the blood, the sedimentation rate of erythrocytes is 64 mm/h. Subacute thyroiditis was diagnosed. The cause of thyrotoxicosis in subacute thyroiditis is:

Гіперпродукція ТТГ у відповідь на запальні зміни в щитоподібній залозі Hyperproduction of TSH in response to inflammatory changes in the thyroid gland

Вироблення антитіл до рецепторів ТТГ Production of antibodies to TSH receptors

Руйнування тиреоцитів і вихід вмісту фолікулів в кров’яне русло Destruction of thyrocytes and release of follicle contents into the bloodstream

Гіперпродукція тиреоїдних гормонів щитоподібною залозою Hyperproduction of thyroid hormones by the thyroid gland

Компенсаторна гіперфункція щитоподібної залози у відповідь на запальні зміни Compensatory hyperfunction of the thyroid gland in response to inflammatory changes

171 / 200
на 6-ту добу підвищилася температура тіла до 38o 171. C, з’явилися лихоманка, слабкість. Під час огляду права молочна залоза збільшена в об’ємі за рахунок інфільтрату без чітких контурів у верхньому квадранті, болюча під час пальпації. Який імовірний діагноз? on the 6th day, the body temperature rose to 38o 171. C, fever, weakness appeared. During the examination, the right mammary gland increased in volume by an infiltrate score without clear contours in the upper quadrant, painful during palpation. What is the likely diagnosis?

Флегмонозний мастит Phlegmonous mastitis

Інфільтративний мастит Infiltrative mastitis

Гангренозний мастит Gangrenous mastitis

Гнійний мастит Purulent mastitis

Лактостаз Lactostasis

172 / 200
Хвора 42-х років скаржиться на виражену слабкість, значне схуднення, посилене випадіння волосся, кровоточивість ясен, порушення менструального циклу, 6іль в кістках та м’язах, здуття живота, бурчання, випорожнення 5-10 раз на добу. Кал рідкий, смердючий. Хворіє з дитинства. Об’єктивно: язик вологий, сосочки згладжені, живіт помірно здутий, більше навколо пупка. Дані копрограми: виявлено багато жирних кислот, сполучнотканинні волокна, клітковина. Який діагноз у хворої? A 42-year-old patient complains of pronounced weakness, significant weight loss, increased hair loss, bleeding gums, irregular menstrual cycle, pain in bones and muscles, bloating abdomen, grunting, defecation 5-10 times a day. Feces are liquid, smelly. Sick since childhood. Objectively: the tongue is moist, the papillae are smoothed, the abdomen is moderately swollen, more around the navel. Coprogram data: a lot of fatty acids, connective tissue fibers, fiber. What is the patient's diagnosis?

Хвороба Уіппла Whipple's disease

Хронічний ентерит Chronic enteritis

Хронічний коліт Chronic Colitis

Хвороба Кона Kohn's disease

Неспецифічний виразковий коліт Nonspecific ulcerative colitis

173 / 200
Роділля 32-х років у I періоді пологів з переймами середньої сили. Дана вагітність четверта, дві попередні закінчилися медичним абортом, третя - кесаревим розтином через передлежання плаценти. Раптом у роділлі з’явився сильний біль в животі, слабкість, артеріальний тиск знизився до 80/50 мм рт.ст. З піхви з’явилися помірні кров’янисті виділення. Серцебиття плода не прослуховується, дрібні частини визначаються ліворуч від середньої лінії живота. Родова діяльність припинилася. Який найбільш імовірний діагноз? A 32-year-old woman in labor in the first stage of labor with moderate labor pains. This is her fourth pregnancy, the previous two ended in a medical abortion, and the third in a caesarean section due to placenta previa. Suddenly the woman in labor developed severe abdominal pain, weakness, blood pressure decreased to 80/50 mm Hg. Moderate bloody discharge appeared from the vagina. Fetal heartbeat is not heard, small parts are determined to the left of the midline of the abdomen. Obstetrics activity has stopped. What is the most likely diagnosis?

Клінічно вузький таз Clinically narrow pelvis

Загроза розриву матки. Дистрес плода Threat of uterine rupture. Fetal distress

Розрив матки, що здійснився Uterine rupture that has occurred

Відшарування нормально розташованої плаценти Detachment of a normally located placenta

Дискоординована пологова діяльність Discoordinated birth activity

174 / 200
Хворий 57-ми років скаржиться на відчуття сухості, болючість під час ковтання, частий настирливий кашель, голос хрипкий. Хвороба розвинулася раптово. В ході ларингоскопії - слизова оболонка гортані гіперемована, голосові складки набряклі, в просвіті гортані в’язкий секрет. Який найбільш імовірний діагноз? A 57-year-old patient complains of a feeling of dryness, pain when swallowing, frequent nagging cough, hoarse voice. The disease developed suddenly. During laryngoscopy, the mucous membrane of the larynx she is hyperemic, the vocal folds are swollen, there is a viscous secretion in the lumen of the larynx. What is the most likely diagnosis?

Бронхіальна астма Bronchial asthma

Дифтерія гортані Diphtheria of the larynx

Флегмонозний ларингіт Phlegmonous laryngitis

Гострий ларингіт Acute laryngitis

Гострий стенозувальний ларинготрахеїт Acute stenosing laryngotracheitis

175 / 200
Хворий 42-х років зі скаргами на гострий біль в епігастральній ділянці, що почався раптово на тлі повного здоров’я, та підвищення температури тіла до 38,5oC, з позитивним симптомом Щоткіна-Блюмберга, потребує хірургічної медичної допомоги. Який вид хірургічної допомоги за часом може бути наданий хворому? A 42-year-old patient with complaints of acute pain in the epigastric area, which began suddenly against the background of complete health, and an increase in body temperature to 38.5oC, with a positive Stotkin-Blumberg symptom, needs surgical medical care. What type of surgical care can be provided to the patient in time?

Планова Plan

Відтермінована або планова Deferred or planned

Екстрена (невідкладна) або планова Emergency (urgent) or planned

Екстрена (невідкладна) Emergency (urgent)

Екстрена (невідкладна) або відтерміно-вана Emergency (urgent) or deferred

176 / 200
У хворого, оперованого з приводу гострого парапроктиту, протягом 5-ти діб на тлі комплексної терапії, при позитивній динаміці місцевого перебігу захворювання, спостерігаються остуди, гіпертермія, тахікардія, ейфорія. Лікар припустив, що у хворого розвинувся сепсис. Яке дослідження може підтвердити діагноз? In a patient operated on for acute paraproctitis, chills, hyperthermia, tachycardia are observed for 5 days against the background of complex therapy, with positive dynamics of the local course of the disease. euphoria. The doctor assumed that the patient developed sepsis. What research can confirm the diagnosis?

Посів крові на наявність збудника Blood culture for pathogen

Визначення ступеню мікробної контамінації рани Determining the degree of microbial contamination of the wound

Рентгенографія легень X-ray lung

Визначення рівня молекул середньої маси Determining the level of average mass molecules

Ультразвукове дослідження печінки Ultrasound examination of the liver

177 / 200
У породіллі 25-ти років відбулися пологи II, термінові, нормальні. 3-я доба післяпологового періоду. Температура тіла - 36,8oC, пульс - 72/хв., артеріальний тиск - 120/80 мм рт.ст. Молочні залози помірно нагрублі, соски чисті. Живіт м’який, безболісний, дно матки на 3 поперечні пальця нижче від пупка. Лохії кров’янисті, помірні. Який імовірний діагноз? A 25-year-old woman gave birth II, urgent, normal. The 3rd day of the postpartum period. Body temperature - 36.8oC, heart rate - 72/min ., blood pressure - 120/80 mm Hg. The mammary glands are moderately thickened, the nipples are clean. The abdomen is soft, painless, the bottom of the uterus is 3 transverse fingers below the navel. Lochia is bloody, moderate. What is the probable diagnosis?

Лактостаз Lactostasis

Залишки плацентарної тканини після пологів Remains of placental tissue after childbirth

Фізіологічний перебіг післяпологового періоду Physiological course of the postpartum period

Післяпологовий метроендометрит Postpartum metroendometritis

Субінволюція матки Subinvolution of uterus

178 / 200
В ході обстеження дитини 5-ти років зі скаргами на постійний кашель з виділенням гнійного мокротиння та вологі хрипи справа у задньонижніх відділах виявлено: гнійний ендобронхіт II-III ступеню, переважно справа, хлориди поту - 36 ммоль/л, на КТ - циліндричні бронхоектази в S9 та S10. Запропонуйте оптимальний метод лікування даної дитини: During the examination of a 5-year-old child with complaints of a constant cough with purulent sputum discharge and wet wheezing on the right in the lower back, it was found: purulent endobronchitis II-III degree, mainly on the right, sweat chlorides - 36 mmol/l, on CT - cylindrical bronchiectasis in S9 and S10. Suggest the optimal method of treatment for this child:

Генно-інженерна терапія Genetic engineering therapy

Кінезотерапія Kinesiotherapy

Видалення уражених сегментів Delete affected segments

Лаваж бронхіального дерева Bronchial tree lavage

Тривала антибіотикотерапія Long-term antibiotic therapy

179 / 200
Мати дитини 1-го року скаржиться на постійний нав’язливий, частий, малопродуктивний кашель, іноді до блювання. Під час об’єктивного дослідження у пацієнта спостерігається прискорене дихання, помірне втягнення нижніх міжреберних м’язів, збільшення передньозаднього розміру грудної клітки. Під час респіраторних інфекцій з’являється бронхообструкція. У періоді новонародженості переніс меконіальний ілеус. Оберіть першочергове обстеження: The mother of a 1-year-old child complains of constant intrusive, frequent, low-productive cough, sometimes leading to vomiting. During an objective examination, the patient has rapid breathing , moderate retraction of the lower intercostal muscles, an increase in the anteroposterior size of the chest. Bronchoobstruction appears during respiratory infections. Meconium ileus was suffered as a newborn. Choose the primary examination:

Рентгенографія органiв грудної клітки X-ray of chest organs

КТ легень CT lung

Дослідження хлоридів поту Sweat Chloride Study

Дослщження на хламідії та мікоплазму Examination of chlamydia and mycoplasma

Генетичне тестування Genetic testing

180 / 200
Хворий 55-ти років скаржиться на постійний біль у попереку, кістках тазу, загальну слабкість, пітливість, схуднення. Має апетит, хворіє протягом 6 місяців. Ro-досліжнення кісток тазу: виражений остеопороз, множинні округлі деструкції до 1 см в діаметрі з чіткими контурами. Аналіз крові: еритроцити - 2,7*1012/л, швидкість осідання еритроцитів - 70 мм/год, тромбоцити - 120*109 /л, загальний білок -110 г/л. У сечі: білок - 7 г/л. Який попередній діагноз? A 55-year-old patient complains of constant pain in the lower back, pelvic bones, general weakness, sweating, weight loss. He has an appetite, has been ill for 6 months. Ro-research pelvic bones: pronounced osteoporosis, multiple rounded destructions up to 1 cm in diameter with clear contours. Blood analysis: erythrocytes - 2.7*1012/l, erythrocyte sedimentation rate - 70 mm/h, platelets - 120*109 /l, total protein -110 g/l. In urine: protein - 7 g/l. What is the previous diagnosis?

Рак шлунку з метастазами Gastric cancer with metastases

Хронічний попереково-крижовий радикуліт Chronic lumbosacral sciatica

Хвороба Бєхтєрєва Bekhterev's disease

Мієломна хвороба Myeloma

Хронічний гломерулонефрит, нефроти-чний синдром Chronic glomerulonephritis, nephrotic syndrome

181 / 200
Хвора 25-ти років скаржиться на наявність декількох щільних, болючих вузлів у правій пахвовій ділянці, підвищення температури тіла до 38°С, загальну слабкість. Хворіє 3 доби. Локально: в правій пахвовій області є три вузлоподібних утворення, які підвищуються над поверхнею шкіри, розмірами від 0,5 см до 1,5 см в діаметрі, різко болючі при пальпації, з чіткими контурами, шкіра над ними багрово-синюшного відтінку, підняття верхньої кінцівки обмежено через біль. Поставте діагноз: A 25-year-old patient complains of the presence of several dense, painful nodes in the right armpit, an increase in body temperature to 38°C, general weakness. She has been ill for 3 days. Locally: in the right axillary region there are three nodular formations that rise above the surface of the skin, the size of which is from 0.5 cm to 1.5 cm in diameter, sharply painful when palpated, with clear contours, the skin above them has a purplish-bluish tint, raising the upper limbs are limited due to pain. Diagnose:

Гнійний гідраденіт Suppurative hidradenitis

Гнійний лімфаденіт Suppurative lymphadenitis

Фурункул Furnish

Карбункул Carbuncle

Абсцес Abscess

182 / 200
До невропатолога звернувся хворий 54-х років зі скаргами на тремор рук, головний біль, металевий присмак у роті, безсоння, підсилення слиновиділення. З анамнезу відомо, що він більше 10 років працює на заводі люмінесцентних ламп. Яким буде ваш діагноз? A 54-year-old patient turned to a neurologist with complaints of hand tremors, headache, metallic taste in the mouth, insomnia, increased salivation. From the anamnesis, it is known that he has been working at a fluorescent lamp factory for more than 10 years. What will be your diagnosis?

Інтоксикація бензолом Benzene intoxication

!нтоксикація марганцем !manganese poisoning

Хронічна інтоксикація ртуттю Chronic mercury intoxication

Хронічна інтоксикація нітрофарбами Chronic intoxication with nitropaints

Хронічна інтоксикація свинцем Chronic lead intoxication

183 / 200
Хлопчику 2 роки, маса тіла - 9,0 кг, зріст - 80 см, обвід голови - 45 см. Значення маси тіла, зросту, обводу голови перебувають між ”-2” та ”-3” стандартними відхиленнями. Визначте рівень фізичного розвитку дитини: The boy is 2 years old, body weight - 9.0 kg, height - 80 cm, head circumference - 45 cm. The values of body weight, height, head circumference are between '-2' and '-3' are standard deviations. Determine the level of physical development of the child:

Дуже високий Very high

Низький Low

Високий High

Середній Average

Дуже низький Very low

184 / 200
Пацієнтка 26-ти років скаржиться на дратівливість, плаксивість, головний біль, нагрубання та болючість молочних залоз. Симптоми виникають за 5-6 днів до менструації та зникають в перший день. Вагінально: матка та придатки без змін. Який найбільш імовірний діагноз? A 26-year-old patient complains of irritability, tearfulness, headache, swelling and tenderness of the mammary glands. Symptoms appear 5-6 days before menstruation and disappear on the first day. Vaginal: uterus and appendages unchanged. What is the most likely diagnosis?

Невроз Neurosis

Альгодисменорея Algodysmenorrhoea

Генітальний ендометріоз Genital endometriosis

Мастопатія Mastopathy

Передменструальний синдром Premenstrual syndrome

185 / 200
Бригада екстреної медичної допомоги доставила пацієнта з гострим інфарктом міокарду до обласного кардіологічного центру, де було виконано кардіохірургічне втручання. В межах якого виду медичної допомоги були надані вказані медичні послуги? The emergency medical care team transported a patient with an acute myocardial infarction to the regional cardiology center, where a cardiac surgical intervention was performed. Within what type of medical care were the specified medical services provided?'

Високоспеціалізована медична допомога Highly specialized medical care

Паліативна медична допомога Palliative medical care

Спеціалізована медична допомога Specialized medical care

Кваліфікована медична допомога Qualified medical care

Екстрена медична допомога Emergency medical assistance

186 / 200
Пацієнт 67-ми років хворіє на КС, стабільну стенокардію напруги протягом 6 років. Об’єктивно: загальний стан задовільний. Частота дихання - 16/хв. В легенях в нижніх відділах застійні хрипи. ЕКГ: миготлива аритмія, гіпертрофія та перенавантаження ЛШ, без вогнищевих змін. Частота серцевих скорочень - 96/хв., артеріальний тиск - 156/92 мм рт.ст. Печінка +2,0 см. Помірні набряки на гомілках. Показники: тропонін Т - 0,08 нг/мл, глюкоза натще - 7,2 ммоль/л. Який рекомендований рівень МНВ (міжнародного нормалізованого відношення) в крові для профілактики тромбоемболічних ускладнень? A 67-year-old patient has been suffering from coronary artery disease, stable angina pectoris for 6 years. Objectively: the general condition is satisfactory. The respiratory rate is 16/min. In the lungs in the lower parts, congestive rales. ECG: atrial fibrillation, LV hypertrophy and overload, without focal changes. Heart rate - 96/min., blood pressure - 156/92 mm Hg. Liver +2.0 cm. Moderate edema on shins. Indicators: troponin T - 0.08 ng/ml, fasting glucose - 7.2 mmol/l. What is the recommended level of INR (international normalized ratio) in the blood for the prevention of thromboembolic complications?

0,8-1,2 0.8-1.2

1,0-1,5 1.0-1.5

2,0-3,0 2.0-3.0

1,0-2,0 1.0-2.0

1,2-1,5 1.2-1.5

187 / 200
У хворого в ході обстеження з приводу болю в поперековій ділянці, періодичного підвищення артеріального тиску виявлено: при УЗД права нирка - 7,3х3,2 см, товщина паренхіми - 0,6 см. Структура нирки не змінена. На екскреторних урограмах права нирка зменшена у розмірах, ЧМС не змінена, функція нирки не зменшена. Патологій з боку лівої нирки не знайдено. Який найбільш імовірний діагноз? In the course of the patient's examination for pain in the lumbar region, periodic increase in blood pressure was found: during ultrasound, the right kidney - 7.3x3.2 cm, parenchyma thickness - 0.6 cm. The structure of the kidney is not changed. On the excretory urograms, the right kidney is reduced in size, CMS is not changed, kidney function is not reduced. No pathologies were found on the left kidney. What is the most likely diagnosis?

Аплазiя правої нирки Aplasia of the right kidney

Подвоєння правої нирки Right kidney doubling

Солітарна кіста правої нирки Solitary right kidney cyst

Рак правої нирки Right kidney cancer

Гіпоплазія правої нирки Right kidney hypoplasia

188 / 200
Дільничний педiатр оглянув 1-місячну дитину, яка народилася від I-ї фізіологічної вагітності, в ході нормальних пологів, з масою - 3 400 г. Перебуває на грудному вигодовуванні, за 1-й місяць набрала 750 г Лікар призначив специфічну профілактику рахіту. Який препарат він обрав? The district pediatrician examined a 1-month-old child who was born from the 1st physiological pregnancy, during a normal delivery, weighing 3,400 g. He is breastfeeding , gained 750 g in the 1st month. The doctor prescribed a specific prevention of rickets. What drug did he choose?

3,44% розчин ретинолу ацетату 3.44% retinol acetate solution

5% розчин токоферолу ацетату 5% tocopherol acetate solution

0,0625% розчин ергокальциферолу 0.0625% ergocalciferol solution

0,125% розчин ергокальциферолу 0.125% ergocalciferol solution

0,125% розчин холекальциферолу 0.125% cholecalciferol solution

189 / 200
Роділлю прийнято на треті пологи в II періоді. Зріст - 172 см, маса - 67 кг. В анамнезі двоє нормальних пологів, три артифіціальних аборти. Народила хлопчика вагою - 3600,0 г. Через 15 хвилин після народження плода розпочалася кровотеча, досягла 380 мл і не припиняється. Ознак відшарування плаценти немає. Яким буде діагноз? The mother was admitted for the third delivery in the second period. Height - 172 cm, weight - 67 kg. In the history of two normal deliveries, three artificial abortions. She gave birth to a boy weighing - 3600.0 g. 15 minutes after the birth of the fetus, bleeding began, reached 380 ml and does not stop. There are no signs of placental abruption. What will be the diagnosis?

Пологи треті, термінові, кровотеча в II періоді пологів Third birth, urgent, bleeding in the second period of labor

Пологи треті, фізіологічний перебіг III періоду Third birth, physiological course of the III period

Пологи треті ,термінові, кровотеча в III періоді пологів Third delivery, urgent, bleeding in the III period of childbirth

Пологи треті, кровотеча у пізньому післяпологовому періоді Third birth, bleeding in the late postpartum period

Пологи треті , кровотеча у ранньому післяпологовому періоді Third birth, bleeding in the early postpartum period

190 / 200
Хворий 36-ти років звернувся до сімейного лікаря зі скаргами на задишку, пришвидшене серцебиття, швидку втому, кровохаркання. Об’єктивно: на щоках рум’янець із ціанотичним відтінком. Під час аускультації: над верхівкою серця посилений I тон, акцент II тону над легеневою артерією, пресистолічний шум над верхівкою; частота серцевих скорочень - 80/хв., ритм регулярний, артеріальний тиск - 100/60 мм рт.ст. Лікар направив хворого на ехокардіографічне дослідження. Яким буде попередній діагноз? A 36-year-old patient turned to the family doctor with complaints of shortness of breath, rapid heartbeat, rapid fatigue, hemoptysis. Objectively: a blush on the cheeks with cyanotic During auscultation: increased I sound over the top of the heart, emphasis of the II sound over the pulmonary artery, presystolic murmur over the top, heart rate - 80/min, regular rhythm, blood pressure - 100/60 mm Hg. The doctor directed patient for echocardiographic examination. What will be the preliminary diagnosis?

Мітральний стеноз Mitral stenosis

Трикуспідальна недостатність Tricuspid insufficiency

Вроджена вада серця Congenital heart defect

Мітральна вада з переважанням стенозу Mitral defect with predominance of stenosis

Аортальна недостатність Aortic insufficiency

191 / 200
Хвора 53-х років скаржиться на гострий біль в правому підребер’ї, який виник через 2 години після вечері, нудоту, блювання жовчю. Хворіє 18 годин. Хвора неспокійна, метушиться в ліжку. Пульс -98/хв., язик обкладений білими нашаруваннями, вологий. Температура тіла - 38,2oC. Під час огляду: права половина відстає в акті дихання, пальпаторно в правому підребер’ї напруження м’язів, болючість. Позитивні френікуссимптом та симптом Грекова-Ортнера. Симптоми подразнення очеревини негативні. Поставте діагноз: A 53-year-old patient complains of acute pain in the right hypochondrium, which occurred 2 hours after dinner, nausea, vomiting bile. She has been ill for 18 hours. The patient restless, fidgeting in bed. Pulse -98/min., tongue covered with white layers, moist. Body temperature - 38.2oC. During examination: the right half lags behind in the act of breathing, palpable muscle tension in the right hypochondrium, tenderness . Phrenicus symptom and Grekov-Ortner symptom are positive. Peritoneal irritation symptoms are negative. Make a diagnosis:

Гострий панкреатит Acute pancreatitis

Гострий апендицит Acute appendicitis

Гострий холецистит Acute cholecystitis

Кишкова непрохідність Intestinal obstruction

Гострий перитоніт Acute peritonitis

192 / 200
Хворий 38-ми років, водій автобуса, у робочий час внаслідок автоаварії отримав важку травму спинного мозку (є акт про нещасний випадок на виробництві). В результаті цього його визнано інвалідом першої групи. Визначити причину інвалідності: A 38-year-old patient, a bus driver, suffered a severe spinal cord injury during working hours as a result of a car accident (there is an act on an accident at work). As a result, his recognized as a disabled person of the first group. Determine the cause of disability:

Важкість травми Injury Severity

Трудове каліцтво Occupational disability

Професійне захворювання Occupational disease

Загальне захворювання General disease

Дорожньо-транспортна пригода Traffic accident

193 / 200
Сімейний лікар, що працює в сімейній амбулаторії, має на обслуговуванні родину з онкохворим пацієнтом. В період епідемії грипу всі члени родини захворіли, а в пацієнта грип ускладнився пневмонією; виникла необхідність постійної систематичної терапії та знеболення. Яке рішення повинен прийняти сімейний лікар? A family doctor working in a family outpatient clinic has a family with a patient with cancer. During the flu epidemic, all members of the family fell ill, and the patient's flu was complicated by pneumonia ; there is a need for constant systematic therapy and analgesia. What decision should the family doctor make?

Направити пацієнта до інфекційного відділення лікарні Send the patient to the infectious department of the hospital

Організувати домашній стаціонар Organize home hospital

Призначити пацієнтові вакцинацію Prescribe vaccination to the patient

Направити пацієнта до паліативного відділення лікарні або до хоспісу Send the patient to the palliative care unit of the hospital or to the hospice

Викликати лікаря-інфекціоніста для консультації Call an infectious disease doctor for a consultation

194 / 200
У хворого 56-ти років в ході ультразвукового обстеження встановлена наявність вегетацій на штучному мітральному клапані, який було імплантовано 3 роки тому, та підклапанних структурах з ознаками деструкції вегетацій та розвитку недостатності штучного клапану. Яке лікування показане хворому? In a 56-year-old patient, during an ultrasound examination, the presence of vegetations on the artificial mitral valve, which was implanted 3 years ago, and subvalvular structures with signs of vegetation destruction and development of artificial valve insufficiency. What treatment is indicated for the patient?

Імплантація штучного водія ритму Implantation of an artificial pacemaker

Призначення вазодилятаторів Prescription of vasodilators

Оперативне (репротезування) Operative (reprosthetics)

Обмеження фізичного навантаження Physical load limitation

Подальше спостереження Follow-up

195 / 200
Хвора скаржиться на схуднення, біль в нижній частині живота, випорожнення з домішками слизу та крові до 15 разів на добу. Під час фіброколоноскопії в ділянці сигмовидної кишки виявлені локальні псевдополіпозні розростання, пласкі поверхневі виразкові ділянки неправильної форми, що не зливаються, вкриті слизом та фібрином; контактна кровоточивість. Який діагноз найбільш імовірний? The patient complains of weight loss, pain in the lower abdomen, stools with impurities of mucus and blood up to 15 times a day. Local pseudopolyposis were detected during fibrocolonoscopy in the area of the sigmoid colon proliferation, flat superficial ulcerated areas of irregular shape that do not coalesce, covered with mucus and fibrin; contact bleeding. What is the most likely diagnosis?

Синдром подразненої кишки Irritable bowel syndrome

Неспецифічний виразковий коліт Nonspecific ulcerative colitis

Хвороба Крона Crohn's disease

Псевдомембранозний коліт Pseudomembranous colitis

Поліпоз кишківника Intestinal polyposis

196 / 200
Хворий 45-ти років скаржиться на нічні напади ядухи до 2-4 разів на місяць, відчуття експіраторної задишки під час фізичного навантаження. В анамнезі - сезонний поліноз впродовж 20-ти років. При проведенні спірометрії отримані наступні дані: ОФВ1 - 62%, ФЖЄЛ - 70% від належних. Зворотність бронхообструкції - 17,9%. Які препарати для базисної терапії необхідно призначити даному хворому? A 45-year-old patient complains of nocturnal attacks of hay fever up to 2-4 times a month, a feeling of expiratory shortness of breath during physical exertion. He has a history of seasonal hay fever for 20 years old. During spirometry, the following data were obtained: FEV1 - 62%, FJEL - 70% of the appropriate ones. Bronchoobstruction reversibility - 17.9%. What drugs for basic therapy should be prescribed for this patient?

Інгаляційні глюкокортикоїди Inhaled glucocorticoids

Бронхолітики короткої дії Broncholytics of short action

Кромони Kromoni

Системні глюкокортикоїди Systemic glucocorticoids

Модифікатори лейкотрієнів Leukotriene modifiers

197 / 200
У хворої 32-х років після переохолодження розвинулися біль у нижніх відділах живота, часте та болісне сечовипускання, субфебрильна температура тіла. В аналізі сечі: білок - 0,3 г/л, лейкоцити в полі зору, еритроцити - 12-15 в полі зору. Яке захворювання можна припустити? A 32-year-old patient developed lower abdominal pain, frequent and painful urination, subfebrile body temperature after hypothermia. Urinalysis: protein - 0.3 g/l, leukocytes in the field of vision, erythrocytes - 12-15 in the field of vision. What disease can be assumed?

Гострий аднексит Acute adnexitis

Гострий цистит Acute cystitis

Позаматкова вагітність Ectopic pregnancy

Гострий пієлонефрит Acute pyelonephritis

Гострий апендицит Acute appendicitis

198 / 200
Хворий 38-ми років скаржиться на біль у горлі, задуху. Захворів після вживання холодного молока. Дихання шумне, частота дихальних рухів - 28/хв., температура тіла - 39oC. Непряма ларингоскопія: набряк та гіперемія надгортанника, голосову щілину не видно. Який терміновий хірургічний захід необхідно вжити? A 38-year-old patient complains of a sore throat, suffocation. He fell ill after drinking cold milk. Breathing is noisy, respiratory rate - 28/min, body temperature - 39oC. Indirect laryngoscopy: swelling and hyperemia of the epiglottis, the glottis is not visible. What urgent surgical measure should be taken?

Інтубація Intubation

Пункція трахеї Trache puncture

Конікотомія Conicotomy

Трахеостомія Tracheostomy

Штучне дихання Artificial respiration

199 / 200
Хворий 70-ти років звернувся зі скаргами на блювоту з’їденим, схуднення, відсутність апетиту. При огляді тургор шкіри знижений, в лівій надключичній ділянці щільний лімфовузол діаметром 1 см. При пальпації живота визначається перерозтя-гнутий шлунок, в епігастрії пальпується пухлиноподібне утворення. Яка патологія зумовлює таку картину? A 70-year-old patient complained of vomiting after eating, weight loss, lack of appetite. On examination, skin turgor is reduced, in the left supraclavicular area there is a dense lymph node with a diameter of 1 see. Palpation of the abdomen reveals an overstretched stomach, a tumor-like formation is palpated in the epigastrium. What pathology causes such a picture?

Дуоденальна виразка Duodenal ulcer

Рак шлунка Stomach cancer

Панкреатит Pancreatitis

Гастрит Gastritis

Виразкова хвороба шлунка Gastric ulcer disease

200 / 200
Під час падіння літака на авіашоу серед потерпілих виявилися дорослі та діти з механічними, термічними та комбінованими ураженнями. Всі були у стані психічного стресу. На полі в перші хвилини після катастрофи перебували лише дві бригади швидкої допомоги, інші бригади під’їжджали до автомобільної стоянки. Якій категорії постраждалих в першу чергу слід було надати першу медичну допомогу? During the fall of the plane at the air show, among the victims were adults and children with mechanical, thermal and combined injuries. All were in a state of mental stress. On the field in the first minutes after there were only two ambulance crews at the scene of the accident, the other crews were approaching the car park. Which category of victims should first of all be given first aid?

Особам похилого віку у важкому стані The elderly are in serious condition

Особам похилого віку незалежно від стану Seniors regardless of status

Дітям незалежно від важкості стану To children regardless of the severity of the condition

Дітям у важкому стані Children are in dire straits

Особам незалежно від віку у важкому стані Persons regardless of age in serious condition